Nurs3100 exam 3

अब Quizwiz के साथ अपने होमवर्क और परीक्षाओं को एस करें!

A nurse is caring for a client who has a new prescription for amphetamine sulfate. The nurse should monitor the client for which of the following adverse effects? -hypotension -tinnitus -tachycardia -bronchospasm

-tachycardia

A nurse is providing teaching for a client who has a new diagnosis of hypertension and a prescription for a low-sodium diet. Which of the following client statements indicate an understanding of the teaching? (Select all that apply.) "I know that deli meats are usually high in sodium." "I need to read food labels when grocery shopping." "I should select organic canned vegetables." "I can refer to the American Heart Association's website for dietary guidelines." "I will stop eating frozen dinners for lunch at work."

"I know that deli meats are usually high in sodium." "I need to read food labels when grocery shopping." "I can refer to the American Heart Association's website for dietary guidelines." "I will stop eating frozen dinners for lunch at work."

A nurse if preparing to administer sumatriptan to a client for the first time. The nurse should instruct the client that sumatriptan is indicated for which of the following conditions? -tonic-clonic seizures -presence of a migraine headache -exacerbation of multiple sclerosis -Alzheimer's disease

-presence of a migraine headache

A nurse is teaching a client who is receiving methohexital sodium. The nurse should monitor the client for which of the following adverse effects? -cardiac excitability -respiratory depression -hyperthermia -hypertension

-respiratory depression

a nurse is caring for a group of postoperative clients. The nurse should identify that morphine is contraindicated for which of the following clients? -a client who had a mastectomy -a client who had a knee arthroplasty -a client who had a colectomy -a client who had a cholecystectomy

-a client who had a cholecystectomy

a nurse is caring for a client who takes low-dose aspirin to prevent cardiovascular events. The client asks the nurse about taking ibuprofen to treat rheumatoid arthritis. Which of the following responses should the nurse make? "ibuprofen will increase your risk for developing salicylism" "ibuprofen will reduce the cardioprotective effects of low-dose aspirin" "low-dose aspirin will reduce the anti-inflammatory effects of ibuprofen" "low-dose aspirin will reduce the analgesic efforts of ibuprofen"

"ibuprofen will reduce the cardioprotective effects of low-dose aspirin"

A nurse is caring for several clients in an extended care facility. Which of the following clients is of the highest priority to observe during meals? -A client who has Parkinson's disease. -A client who has decreased vision. -A client who has poor dentition. -A client who has anorexia.

-A client who has Parkinson's disease.

A nurse is providing teaching to a client about instilling pilocarpine for managing open-angle glaucoma. Which of the following instructions should the nurse include in the teaching? (Select all that apply.) -Apply gentle pressure to the nasolacrimal duct for 1 min after instilling the drops -Do not touch the tip of the dropper -Wash hangs after instilling the drops -Rub eyes gently after instilling the drops -Remove contact lenses prior to instilling the drops

-Apply gentle pressure to the nasolacrimal duct for 1 min after instilling the drops -Do not touch the tip of the dropper -Remove contact lenses prior to instilling the drops

A nurse is providing teaching to a client who is about to begin amitriptyline therapy to treat major depressive disorder. Which of the following instructions should the nurse include? (Select all that apply.) -Change positions slowly from sitting or lying to standing -Do not stop taking the drug abruptly -Avoid crushing the tablet -Take the drug at bedtime to prevent daytime drowsiness -Increase fiber and fluid intake

-Change positions slowly from sitting or lying to standing -Do not stop taking the drug abruptly -Take the drug at bedtime to prevent daytime drowsiness -Increase fiber and fluid intake

A nurse is caring for a client who has a new prescription for pramipexole to treat Parkinson's disease. The nurse should recognize that which of the following lab tests requires monitoring? -C-reactive protein -Creatinine clearance -Thyroid function -CBC

-Creatinine clearance

A nurse is planning care for a client who has mechanical fixation of the jaw following a motorcycle crash. Which of the following actions should the nurse include in the plan of care? Select all that apply. -Encourage intake of fluids between meals. -Thicken liquids to honey consistency -Assist the client to use a straw to drink liquids -Educate the client about the use of a nasogastric tube. -Ensure that the client receives ground meats

-Encourage intake of fluids between meals. -Assist the client to use a straw to drink liquids

A nurse is caring for a client who is transitioning to an oral diet following a partial laryngectomy. Which of the following actions should the nurse take to reduce the client's risk for aspiration? -Instruct the client to follow each bite of food with a drink of water. -Consult the dietitian about providing the client with a thin liquid diet -Encourage the client to tuck the chin when swallowing -Request to have the client's oral medications provided in liquid form.

-Encourage the client to tuck the chin when swallowing

A nurse is administering fentanyl to a client to reduce pain. Which of the following drugs should the nurse have available to reverse the effects of fentanyl? -Neostigmine -Succinylcholine -Naloxone -Dantrolene

-Naloxone

A nurse is reviewing the medical record of a client who has a newly diagnosed seizure disorder and a new prescription for valproic acid and phenytoin. The nurse should identify that which of the following can occur as a result of an interaction between these drugs? -Hyperammonemia -Phenytoin toxicity -Hypertension -Peptic ulcer disease

-Phenytoin toxicity

A nurse is planning care for a client who is receiving treatment for malnutrition. The client is scheduled for discharge to their home where they live alone. Which of the following actions should the nurse include in the plan of care? (Select all that apply.) -Recommend drinking a supplement between meals. -Advise the client to purchase frozen fruits and vegetables. -Educate the client on how to read nutrition labels -Encourage the client to purchase nonperishable boxed meals. -Consult social services to arrange home meal delivery.

-Recommend drinking a supplement between meals. -Advise the client to purchase frozen fruits and vegetables. -Educate the client on how to read nutrition labels -Consult social services to arrange home meal delivery.

When reviewing the indications for various antidepressants, a nurse should identify that bupropion hydrochloride is an appropriate choice for clients who have which of the following? (Select all that apply.) -Motion sickness -Seasonal affective disorder -Insomnia -Nicotine addition -Depression

-Seasonal affective disorder -Nicotine addition -Depression

The nurse educator is reviewing the process of synaptic transmission following the sympathetic pathway. In which order will the nurse explain the steps of synaptic​ transmission?

-action potential travels across the preganglionic neuron -action potential encounters cholinergic receptors -action potential travels across the postganglionic neuron -action potential encounters adrenergic receptors -target tissue is reached

A nurse is teaching a client who received a prescription for interferon beta-1a for the treatment of multiple sclerosis. Which of the following information should the nurse include? -have kidney function tests done every month for a year -take an extra dose in muscle aches occur -store the drug at a room temperature after mixing it -administer the drug in your thigh or upper arm

-administer the drug in your thigh or upper arm

A nurse is teaching a client who has a prescription for baclofen. Which of the following instructions should the nurse include? -avoid driving until the drug's effects are evident -stop taking the med immediately if headache occurs -take the drug as needed for spasticity -take the drug with antacids to reduce gastric effects

-avoid driving until the drug's effects are evident

A nurse is teaching a client who has Alzheimer's disease and their caregiver about memantine. Which of the following instructions should the nurse include? -increase fluid intake to improve renal excretion -report memory loss or confusion -watch for signs of liver impairment, such as jaundice & abdominal pain -avoid taking OTC antiacids

-avoid taking OTC antiacids

A nurse is teaching the family of a client who has a new diagnosis of Alzheimer's disease about donepezil. Which of the following information should the nurse include? -monitor for constipation -the dosage will be increased weekly to provide optimum therapeutic effect -administering the drug first thing in the morning promotes effectiveness -avoid the use of NSAIDs for pain

-avoid the use of NSAIDs for pain

A nurse is teaching a client who has a prescription for carbamazepine. Which of the following instructions should the nurse include to help the client avoid adverse effects of this drug? -begin taking the drug at a low dosage -discontinue the drug immediately if diarrhea occurs -have serum glucose levels checked regularly -take the drug on an empty stomach

-begin taking the drug at a low dosage

A nurse is teaching a client who is taking levodopa/carbidopa to treat Parkinson's disease. Which of the following instructions should the nurse include? -change position slowly to prevent orthostatic hypotension -eat a high-protein snack to increase absorption -take the drug at bedtime to avoid daytime drowsiness -expect eye twitching to develop with long-term therapy

-change position slowly to prevent orthostatic hypotension

A nurse is teaching a client who is about to begin sumatriptan therapy to treat migraine headaches. The nurse should instruct the client to monitor for which of the following adverse effects? -chest pain -polyuria -joint pain -insomnia

-chest pain

A nurse is preparing to administer memantine to a client who has Alzheimer's disease. Which of the following findings in the client's medical history indicates a need to withhold the drug and notify the provider? -pancreatic cancer -hypotension -cirrhosis -osteoporosis

-cirrhosis

A nurse is caring for a client who has a new prescription for dantrolene to treat skeletal muscle spasms. The nurse should instruct the client to report which of the following adverse effects? -slow heart rate -cough -diarrhea -hearing loss

-diarrhea

a nurse is teaching a client who has a new prescription for prednisone. Which of the following instructions should the nurse include? (select all that apply) -reduce the dose during periods of stress -discontinue the drug gradually -report illness or infection -increase intake of calcium and vitamin D -monitor for signs of gastric bleeding

-discontinue the drug gradually -report illness or infection -increase intake of calcium and vitamin D -monitor for signs of gastric bleeding

A nurse is caring for a client who is taking interferon beta-1b. The nurse should identify that which of the following findings indicates a potential serious adverse effect of this drug? -tinnitus -twitching eyelids -blue-green skin discoloration -fatigue

-fatique

A nurse is teaching a client who has a new prescription for valproic acid to treat a seizure disorder. The nurse should instruct the client to monitor for which of the following adverse effects. (select all that apply) -hirsutism -drowsiness -headache -ataxia -rash

-headache -drowsiness -rash

A nurse is teaching a client who has a new diagnosis of Parkinson's disease about how levodopa/carbidopa can help control symptoms. The nurse should identify that the drug has which of the following pharmacologic effects? -increases available acetylcholine in the brain -inhibits norepinephrine metabolism in the brain -inhibits serotonin metabolism in the brain -increases available dopamine in the brain

-increases available dopamine in the brain

A nurse is providing teaching for a client who has a new prescription for valproic acid to treat a seizure disorder. The nurse should instruct the client to monitor for which of the following adverse effects? -hirsutism -depression -jaundice -gum irritation

-jaundice

A nurse is caring for a client who has a new prescription for dantrolene to treat skeletal muscle spasms. The nurse should identify that which of the following lab tests requires monitoring? -serum potassium -liver function -serum sodium -thyroid function

-liver function

A nurse is caring for a client who has been taking selegiline to treat Parkinson's disease. The provider is considering the use of analgesics for the client but should be aware that a drug interaction between selegiline and meperidine can result in which of the following? -frequent urination -jaundice -cellulitis -muscle rigidity

-muscle rigidity

A nurse is caring for a client who is taking donepezil to treat Alzheimer's disease. For which of the following adverse effects should the nurse monitor? -confusion -dry mouth -nausea -double vision

-nausea

A nurse is teaching a client about interferon beta-1a. Which of the following instructions should the nurse give to help the client avoid the adverse effects of this drug? -premedicate with acetaminophen -take the drug with food -increase your fluid intake -take the drug in the morning

-premedicate with acetaminophen

A nurse is teaching a client who has a prescription for modafinil to treat narcolepsy. Which of the following instructions should the nurse include? -take the drug in the morning -take the drug 30 min before bedtime -anticipate daytime drowsiness -expect urinary frequency

-take the drug in the morning

A nurse is caring for a client who has been taking amphetamine/dextroamphetamine sulfate for the treatment of attention deficit hyperactivity disorder (ADHD) for 2 weeks. The nurse should report which of the following findings to the provider? -weight loss of 2.3 kg (5 lbs) -BP 110/70 mm Hg -apical pulse 8-/min -respiratory rate 16/min

-weight loss of 2.3 kg (5 lbs)

ATI "Pain & Inflammation"

...

ATI Neurological systems (Part 2)

...

ATI: The Neurological System (Part 1)

...

MyLab Chapter 12

...

MyLab Chapter 13

...

MyLab Chapter 14

...

MyLab Chapter 15

...

MyLab Chapter 16

...

MyLab Chapter 18

...

MyLab Chapter 19

...

MyLab Chapter 20

...

MyLab Chapter 21

...

MyLab Chapter 22

...

MyLab chapter17

...

Nutrition Chapter 11 quiz

...

A client's heart rate is 72 beats per minute with a regular rhythm. How does the nurse interpret this data in relation to the client's peripheral nervous system? Select all that apply... 1) The sympathetic system is in control. 2) The parasympathetic system is dominating. 3) Both parasympathetic and sympathetic systems are influencing cardiac status. 4) The autonomic nervous system is affecting cardiac status. 5) The somatic nervous system is affecting cardiac status.

2) The parasympathetic system is dominating. 3) Both parasympathetic and sympathetic systems are influencing cardiac status. 4) The autonomic nervous system is affecting cardiac status.

Which should the nurse understand is the primary therapeutic goal for the use of pharmacotherapy in the treatment of a neuromuscular​ disorder? A. Promote independence. B. Stop the​ client's muscle spasms. C. Improve appearance. D. Promote the ease of exercise.

A.

A client admitted tells the nurse he or she has been taking​ "yellow jackets"​ (pentobarbital, [Nembutal]) but have not had any for a few days. Which should the nurse recognize is the highest​ priority? A. Prepare to manage a withdrawal that could be life threatening. B. Prepare to manage the​ client's depression. C. Prepare to do frequent room​ searches, as the​ client's friends will most likely bring drugs in for him or her. D. Prepare to manage the​ client's pain.

A

A client experiencing the effects from marijuana asks the nurse how long they will last. Which response should the nurse​ provide? A. ​"The effects can last up to 24​ hours." B. ​"The effects will last a few​ hours." C. ​"It depends on how much you​ smoked." D. ​"It depends on your body​ weight."

A

A client tells the nurse he or she would like a prescription to help stop smoking. Which should the nurse anticipate the healthcare provider will​ prescribe? A. Bupropion​ (Zyban) B. Naltrexone​ (ReVia, Vivitrol) C. Disulfiram​ (Antabuse) D. Dextroamphetamine​ (Dexedrine)

A

A client tells the nurse​ "I am going to get clean. I​ haven't had any drugs or any alcohol for 2​ days." Which substance should the nurse anticipate can result in life threatening​ withdrawal? A. Alcohol B. Marijuana C. Cocaine D. Heroin

A

A family member of a deceased client that overdosed on cocaine asks the nurse how he or she died. Which should the nurse recognize is the most likely caused the death of the​ client? A. Cardiac arrest B. Drug impurity C. Aneurysm D. Rhabdomyolysis

A

The nurse is assessing a client who has taken LSD. Which assessment finding should the nurse​ anticipate? A. Increased temperature B. Hypotension C. Bradycardia D. Pinpoint pupils

A

The nurse is providing education for a client who desires to stop taking heroin. Which information should the nurse include in the​ teaching? Select all that apply. A. ​"You are probably going to have some withdrawal​ symptoms, but there are medications that can help​ you." B. ​"Expect to remain on methadone for quite a​ while." C. ​"Methadone will produce the same high as the heroin you have been​ using." D. ​"You will need a supply of syringes and needles for your methadone​ treatment." E. ​"Methadone does not cause any withdrawal​ symptoms."

A,B

Which information should the nurse include in the community education for​ marijuana? Select all that apply. A. THC is the ingredient that causes most of the psychoactive effects. B. Metabolites of marijuana can be detected in the urine for several days after last use. C. Metabolites of marijuana can be detected in the body for months to years after last use. D. Smoking marijuana can decrease the risk of glaucoma. E. Smoking marijuana results in a greater accumulation of tar in the lungs than does cigarette smoking.

A,B,C,E

A client tells the nurse he or she has been abusing his or her antianxiety prescription. Which findings should the nurse recognize are related to a psychological​ dependence? Select all that apply. A. The​ patient's history reveals the​ long-term use of high doses of an antianxiety prescription. B. The client describes the​ "need" to use the antianxiety prescription despite not feeling anxious. C. The client describes feeling nauseous with abdominal cramping. D. The client states his or her family does not speak to him or her because of the need for the antianxiety prescription. E. The client states he or she has not slept in days and feel very anxious tired.

A,B,D

A client asks the nurse for information about cannabis infused edibles. Which information should the nurse be prepared to​ discuss? Select all that apply. A. Delayed onset B. Longer duration of action C. Physical dependence D. Higher risk of unintentional overdose E. Large doses are associated with psychedelic high

A,B,D,E

The nurse is preparing to assess a client that is withdrawing from opioids. Which assessment findings should the nurse​ anticipate? Select all that apply. A. Abdominal cramping B. Orthostatic hypotension C. ​Hot, dry skin D. Violent yawning E. Agitation

A,B,D,E

A client asks the nurse what physiological effects occur if he or she quit smoking. Which should the nurse include in the​ education? Select all that apply. A. Headaches B. Depression C. Increased appetite D. Nausea E. Insomnia

A,C,E

A client must eliminate caffeine intake due the contraindication with a newly prescribed medication. Which withdrawal symptoms should the nurse discuss with the​ client? ​Note: Credit will be given only if all correct choices and no incorrect choices are selected. Select all that apply. A. Depression B. Insomnia C. Weight gain D. Urinary retention E. Headache

A,E

A client asks the nurse how skeletal muscle relaxants work. Which response should the nurse provide the​ client? A. ​"They inhibit upper motor neuron activity within the central nervous​ system." B. ​"They increase the amount of neurotransmitter within the​ muscles." C. ​"They stimulate motor activity within the​ brainstem." D. ​"They work primarily by stimulating the peripheral nervous​ system."

A.

A client asks the​ nurse, "Can you tell me how capsaicin works to treat muscle​ pain?" Which statement should the nurse include in the​ response? A. ​"Decreases the sensation of pain by interfering with the chemical​ messengers." B. ​"Decreases the activity of the chemicals that stimulate muscle​ contraction." C. ​"Stimulates the body to produce endorphins that block the pain​ signal." D. ​"Interferes with the muscles ability to isometrically​ contract."

A.

A client has been prescribed dantrolene​ (Dantrium). Which question should the nurse ask the​ client? A. ​"Are you currently​ pregnant?" B. ​"Does you have a history of​ psoriasis?" C. ​"Do you have a history of gastric ulcer​ disease?" D. ​"Do you have a history of gallbladder​ disease?"

A.

The nurse is reviewing the prescription history for a client prescribed dantrolene​ (Dantrium). Which prescription should concern the​ nurse? A. Verapamil​ (Calan) B. Clarithromycin​ (Biaxin) C. Methylphenidate​ (Concerta) D. Insulin

A.

Which is the​ nurse's priority assessment when monitoring a client receiving dantrolene​ (Dantrium) intravenously? A. Intravenous site B. Breath sounds C. Urinary output D. Blood glucose

A.

Which should the nurse include in the plan of care for the client that has received baclofen​ (Lioresal)? A. Fall precautions B. Fluid Restriction C. Intake and Output D. Dietary restrictions

A.

Which assessment finding warrants the nurse to hold the dosage of benztropine​ (Cogentin) and notify the healthcare​ provider? A. A pulse of 112 B. Blood pressure of​ 142/80 mmHg C. A respiratory rate of 14 D. A temperature of ​100.2°F

A. A pulse of 112

A client with an oxycodone overdose has a respiratory rate of 8 and difficult to arouse. Which prescribed treatment should the nurse anticipate​ administering? A. Administering opioid antagonist B. Administering activated charcoal C. Preparing for intubation and mechanical ventilation D. Administering an opioid agonist

A. Administering opioid antagonist

The educator is preparing to discuss the classifications of local anesthetics. Which should the educator include when discussing​ amides? Select all that apply. A. Amides have fewer side effects than esters. B. Amides block potassium entry into the cell. C. Amides are similar in structure to cocaine. D. Amides tend to last longer than esters. E. Amides block calcium entry into the cell.

A. Amides have fewer side effects than esters. D. Amides tend to last longer than esters.

The nurse is preparing to provide client education on degenerative diseases of the nervous system. Which information should the nurse​ include? Select all that apply. A. Amyotrophic lateral sclerosis B. Multiple sclerosis C. ​Alzheimer's disease D. Myasthenia Gravis E. ​Parkinson's disease

A. Amyotrophic lateral sclerosis B. Multiple sclerosis C. ​Alzheimer's disease E. ​Parkinson's disease

A nurse is reading about the development of drugs that inhibit the parasympathetic nervous system. The nurse would look for articles about which drug​ classes? Select all that apply. A. Anticholinergics B. Parasympathomimetics C. Parasympatholytics D. Cholinergics E. Muscarinic blockers

A. Anticholinergics C. Parasympatholytics E. Muscarinic blockers

A client taking risperidone​ (Risperdal) for schizophrenia is experiencing signs of hypotension with certain activities. Which information should the nurse provide the​ client? Select all that apply. A. Avoid being outside in the heat of the day. B. Avoid hot baths or hot showers. C. Avoid vitamin​ C-containing preparations. D. Change positions​ slowly, especially from lying or sitting to standing. E. Take frequent naps during the day.

A. Avoid being outside in the heat of the day. B. Avoid hot baths or hot showers. D. Change positions​ slowly, especially from lying or sitting to standing.

Which client should the nurse recognize a prescription for sumatriptan​ (Imitrex) may be​ beneficial? A. A​ 27-year-old asthmatic male B. A​ 73-year-old with angina pectoris C. A​ 36-year-old female with preeclampsia D. A​ 45-year-old diabetic male

A. A​ 27-year-old asthmatic male

A client asks the nurse which types of therapies are most helpful for depression. Which therapies should the nurse plan to discuss with the​ client? Select all that apply. A. Behavioral therapy B. Interpersonal therapy C. ​Cognitive-behavioral therapy D. Psychodynamic therapy E. Crisis therapy

A. Behavioral therapy B. Interpersonal therapy C. ​Cognitive-behavioral therapy D. Psychodynamic therapy

Which prescriptions should the nurse anticipate to be included in balanced​ anesthesia? Select all that apply. A. Benzodiazepines B. Neuromuscular blockers C. Inhaled anesthetics D. ​Proton-pump inhibitors E. Intravenous anesthetics

A. Benzodiazepines B. Neuromuscular blockers C. Inhaled anesthetics E. Intravenous anesthetics

A client asks the nurse what causes schizophrenia. Which contributing factors should the nurse include in the client​ teaching? Select all that apply. A. Biological brain disorder B. Alcohol use during pregnancy C. Stressful environment D. Genetic link E. Abnormal levels of neurotransmitters in the brain

A. Biological brain disorder D. Genetic link E. Abnormal levels of neurotransmitters in the brain

The nurse is caring for a client with multisystem organ failure. Which client assessment findings are under the control of the sympathetic nervous​ system? Select all that apply. A. Blood glucose level 210​ mg/dL. B. Blood pressure​ 180/90 mmHg. C. Extremities are cool. D. Respiratory rate 14 and regular. E. Hyperactive bowel sounds.

A. Blood glucose level 210​ mg/dL. B. Blood pressure​ 180/90 mmHg. C. Extremities are cool.

A client states to the​ nurse, "I stopped taking that beta blocker last week because it made me so​ tired." Which are the priority nursing​ assessments? Select all that apply. A. Blood pressure B. Heart rhythm C. Urinary output D. Presence of chest pain E. Presence of respiratory crackles

A. Blood pressure B. Heart rhythm D. Presence of chest pain

Which prescription should the nurse recognize would most benefit a client experiencing periods of mania and​ depression? A. Carbamazepine​ (Tegretol) B. Methylphenidate​ (Ritalin) C. Atomoxetine​ (Strattera) D. Amitriptyline​ (Elavil)

A. Carbamazepine​ (Tegretol)

The nurse has administered sumatriptan​ (Imitrex) to a client. Which effects should the nurse monitor the patient​ for? A. Decreased level of consciousness B. Muscle weakness C. Hypotension D. Urinary retention

A. Decreased level of consciousness

Which symptoms should the nurse assess a client for who is at risk for​ mania? Select all that apply. A. Decreased sleep B. Weight loss C. Sluggish activity D. Muscle pain E. Increased talkativeness

A. Decreased sleep B. Weight loss E. Increased talkativeness

While conducting a health history for a client with schizophrenia. Which behaviors associated with positive signs of schizophrenia should the nurse​ anticipate? Select all that apply. A. Disorganized sentence structure B. Kicking the nurse C. Not responding to questions D. Difficulty following instructions E. Stating that God wants the patient to go home

A. Disorganized sentence structure B. Kicking the nurse E. Stating that God wants the patient to go home

Which information should the nurse include in the education for a client prescribed sumatriptan​ (Imitrex)? Select all that apply. A. Do not drive until the effects of the medication are known. B. Avoid pseudoephedrine​ (Sudafed) while taking the prescription. C. Take the prescription with a meal high in protein. D. Take the prescription with food. E. Increase fluid intake.

A. Do not drive until the effects of the medication are known. B. Avoid pseudoephedrine​ (Sudafed) while taking the prescription.

Which effects associated with the prescription of midazolam​ (Versed) should the nurse anticipate when administered as part of surgical​ anesthesia? Select all that apply. A. Effects consistent with those observed with other benzodiazepines B. Reduction of anxiety and stress associated with surgery C. Central nervous system depression and skeletal muscle relaxation D. Maintenance of stable cardiac and respiratory activity E. Absent cardiac dysrhythmias

A. Effects consistent with those observed with other benzodiazepines B. Reduction of anxiety and stress associated with surgery C. Central nervous system depression and skeletal muscle relaxation

A client asks the nurse what causes bipolar disorder. Which neurotransmitters should the nurse include when discussing the manic phase of the​ disorder? Select all that apply. A. Excessive glutamate B. Excessive norepinephrine C. Deficiency of​ gamma-aminobutyric acid D. Deficiency of dopamine E. Excessive serotonin

A. Excessive glutamate B. Excessive norepinephrine C. Deficiency of​ gamma-aminobutyric acid

A client is prescribed a medication that will block muscarinic receptors. The nurse realizes that this medication has implications for which body​ systems? Select all that apply. A. Eyes B. Respiratory C. Cardiac D. Endocrine E. Metabolic

A. Eyes B. Respiratory C. Cardiac

A client with a history of schizophrenia has stopped his or her prescribed clozapine​ (Clozaril) after 4 weeks of therapy. Which should the nurse recognize may have contributed to the client abruptly stopping his or her​ prescription? Select all that apply. A. Feeling sedated B. Significant weight gain C. Agitation D. Fatigue and headache E. Uncontrollable sexual urges

A. Feeling sedated B. Significant weight gain C. Agitation D. Fatigue and headache

A client has been prescribed ketogenic diet to supplement pharmacotherapy for a seizure disorder. Which food items should the nurse encourage the client to​ consume? Select all that apply. A. Foods high in fat B. Salty foods C. Foods high in vitamin K D. Low protein foods E. Low carbohydrate foods

A. Foods high in fat D. Low protein foods E. Low carbohydrate foods

The nurse is preparing to administer chlorpromazine intramuscularly. Which anatomical location should the nurse administer the​ medication? A. Gluteal B. Deltoid C. Vastus lateralis D. Ventrogluteal

A. Gluteal

A client prescribed an antipsychotic tells the nurse he or she not had a bowel movement for 2 days. Which initial action should the nurse​ take? A. Have the client drink prune juice and additional fluids. B. Obtain a prescription for a stool softener. C. Hold the prescription until the client has a bowel movement. D. Request a prescription for a Fleet Enema.

A. Have the client drink prune juice and additional fluids.

A client has sustained a large blood loss. During the​ assessment, the nurse realizes that which findings are under the control of the nervous​ system? Select all that apply. A. Heart rate B. Blood pressure C. Pupil size D. Bowel sounds E. Fluid volume

A. Heart rate B. Blood pressure C. Pupil size D. Bowel sounds

A client who is in heart failure is administered a beta1 agonist. Which assessment finding indicates the prescription is​ effective? Select all that apply. A. Heart rate increases B. Pulse becomes stronger C. Pupils dilate D. Dysrhythmias dissipate E. Blood pressure drops

A. Heart rate increases B. Pulse becomes stronger

A nurse is preparing to provide education on the use of heroin. Which information should the nurse​ include? Select all that apply. A. Heroin is an opioid drug. B. Heroin is a narcotic drug. C. Heroin causes CNS agitation. D. Heroin is the same drug as morphine. E. Heroin is not as dangerous as oxycodone.

A. Heroin is an opioid drug. B. Heroin is a narcotic drug.

The nurse is providing education to a parent of a​ 5-year-old that experienced a febrile seizure. Which information should the nurse​ include? A. High temperatures rarely induce seizures in children. B. A​ five-year-old is too old for febrile seizures. C. Seizure medications are usually prescribed for febrile seizures. D. After a febrile seizure the child returns to consciousness slowly.

A. High temperatures rarely induce seizures in children.

The patient has bipolar disorder and is in a manic phase. The physician prescribes lithium​ (Eskalith). The​ client's current lithium level is 0.4. Which assessment findings should the nurse​ anticipate? A. Hyperactivity and pressured speech. B. A decrease in manic behavior. C. Signs and symptoms of depression. D. A return to baseline behavior.

A. Hyperactivity and pressured speech.

The nurse is preparing to provide instructions on the use of an epinephrine​ auto-injector. Which information should the nurse​ include? Select all that apply. A. If you need to use this​ pen, seek medical advice as​ follow-up. B. You can dispose of a used EpiPen in your regular trash. C. Keep an extra EpiPen on hand. D. Store this device in your refrigerator. E. Carry an EpiPen in your​ car's glovebox.

A. If you need to use this​ pen, seek medical advice as​ follow-up. C. Keep an extra EpiPen on hand.

A client receiving pharmacotherapy for the treatment of​ Parkinson's disease has experienced increased eye twitching tremors. Which treatment interventions should the nurse​ anticipate? Select all that apply. A. Increase in the dosage of the prescription B. Changing the interval between the doses of prescriptions C. Changing the route of administration of the prescription D. A scheduled drug holiday E. Implementing an adjunctive prescription

A. Increase in the dosage of the prescription B. Changing the interval between the doses of prescriptions E. Implementing an adjunctive prescription

A client is suspected to have been overdosed with Propofol​ (diprivan). Which immediate nursing actions are​ indicated? Select all that apply. A. Increase the intravenous fluid rate. B. Monitor the client for development of an allergic skin reaction. C. Monitor the client for respiratory depression. D. Obtain serum calcium levels. E. Administer a benzodiazepine.

A. Increase the intravenous fluid rate. C. Monitor the client for respiratory depression.

Which reflects the primary reason a neuromuscular blocking agent is used for a client receiving general​ anesthesia? A. Induce total skeletal muscle relaxation B. Induce unconsciousness C. Prevent adverse effects associated with inhaled gases D. Potentiate analgesic effects

A. Induce total skeletal muscle relaxation

Which statement describes the purpose for the use of acetylcholinesterase inhibitors prescribed to treat​ Alzheimer's disease? A. Intensify the effect of acetylcholine at the receptor. B. Increase synthesis of acetylcholine. C. Increase enzymatic breakdown leading to increased neuronal production. D. Reverse the structural damage within the brain.

A. Intensify the effect of acetylcholine at the receptor.

The nurse is preparing to administer phenytoin​ (Dilantin). Which route should the nurse anticipate to administer the​ prescription? A. Intravenously B. Intramuscular C. Subcutaneous D. Central venous catheter

A. Intravenously

A client has been prescribed an alpha1 adrenergic agonist drug. The nurse would plan to monitor for effects from which​ organs? Select all that apply. A. Kidneys B. Eyes C. Heart D. Bowels E. Lungs

A. Kidneys B. Eyes D. Bowels E. Lungs

The nurse is reviewing routine medications for a client prescribed phenylephrine​ (Neo-Synephrine). Which medications are most​ concerning? Select all that apply. A. MAO inhibitors B. Tricyclic antidepressants C. Iron supplements D. Digoxin E. Aspirin

A. MAO inhibitors B. Tricyclic antidepressants C. Iron supplements D. Digoxin

Which should the nurse understand are the primary goals for the pharmacological treatment of multiple​ sclerosis? Select all that apply. A. Modifying the progression of the disease. B. Treating acute exacerbations. C. Managing symptoms. D. Curing the disease. E. Remyelinating nerve fibers.

A. Modifying the progression of the disease. B. Treating acute exacerbations. C. Managing symptoms.

A client is being assessed for the presence of pheochromocytoma. Which interventions should the nurse​ implement? Select all that apply. A. Monitoring blood pressure. B. Monitoring bowel sounds. C. Administration of phentolamine​ (Regitine) D. Rapid administration of normal saline. E. Urinary catheterization to bypass prostatic enlargement.

A. Monitoring blood pressure. C. Administration of phentolamine​ (Regitine)

Which classification of prescription used to treat depression works by preventing enzymatic destruction of the neurotransmitter​ norepinephrine? A. Monoamine oxidase inhibitors​ (MAOIs) B. Tricyclic antidepressants​ (TCAs) C. Selective serotonin reuptake inhibitors​ (SSRIs) D. ​Serotonin-norepinephrine reuptake inhibitors​ (SNRIs)

A. Monoamine oxidase inhibitors​ (MAOIs)

The nurse is preparing to asses a client with amyotrophic lateral sclerosis​ (Lou Gehrig's​ disease). Which assessment findings should the nurse​ anticipate? Select all that apply. A. Muscle weakness B. Muscle atrophy C. Mood alterations D. Slow shuffling gait E. Abnormal posture

A. Muscle weakness B. Muscle atrophy

During an​ assessment, the nurse notes that the client prescribed risperidone​ (Risperdal) has a temperature of 102 degrees F. Which should the nurse suspect has​ occurred? A. Neuroleptic malignant syndrome. B. Extrapyramidal side effects. C. Acute urinary infection. D. Bacterial pneumonia.

A. Neuroleptic malignant syndrome.

A nursing instructor is reviewing receptor types with a nursing student. The instructor identifies that learning has occurred when the student responds that which type of receptor is found at the ganglionic synapse of both the sympathetic and parasympathetic nervous​ systems? A. Nicotinic. B. Alpha receptors. C. Beta receptors. D. Muscarinic.

A. Nicotinic.

A client who was administered prazosin​ (Minipress) became unconscious 30 minutes after the first dose. Which prescriptions should the nurse prepare for​ resuscitation? Select all that apply. A. Normal saline B. Dobutamine C. Atenolol​ (Tenormin) D. Carvedilol​ (Coreg) E. Propranolol​ (Inderal)

A. Normal saline B. Dobutamine

The nurse observes a client that stops talking​ mid-sentence and has a blank stare for 5 seconds. Which should the nurse suspect has​ occurred? A. Petit mal seizure B. Status epilepticus C. Drop attack D. Simple partial seizure

A. Petit mal seizure

The nurse is preparing to assess a client with​ Parkinson's disease. Which finding should the nurse recognize interferes with the​ client's ability to control their hand​ movements? Select all that apply. A. Pill rolling B. Tremor C. Stooped posture D. Lack of arm swing E. Difficulty bending the arms

A. Pill rolling B. Tremor

A nursing instructor is teaching student nurses about the sympathetic nervous system. The instructor recognizes that learning has occurred when the students say which of the following responses are regulated by the sympathetic nervous system. A. Pupil dilation. B. Sex organ stimulation. C. Bronchial constriction. D. Stimulates salivation.

A. Pupil dilation.

A client asks the nurse why he or she receiving is preoperative prescriptions. Which information should the nurse include in the​ education? Select all that apply. A. Reduce anxiety and facilitate sedation B. Facilitation of a faster recovery C. Reduce the risk of aspiration pneumonia D. Reduce the risk of a postoperative ileus E. Manage pain

A. Reduce anxiety and facilitate sedation C. Reduce the risk of aspiration pneumonia E. Manage pain

A client tells the nurse they feel depressed during the winter. Which type of depression should the nurse suspect the client is​ experiencing? A. Seasonal affective disorder B. Baby blues C. Bipolar disorder D. ​Obsessive-compulsive disorder

A. Seasonal affective disorder

Which drug category can be used for treating​ anxiety? A. Seizure drugs B. Anticoagulants C. Antibiotics D. Antitussives

A. Seizure drugs

Which existing pharmacological treated condition should the nurse be most concerned about for a client prescribed haloperidol​ (Haldol)? A. Severe depression B. Diabetes mellitus C. Hypothyroidism D. Urinary tract infection

A. Severe depression

A client is prescribed zolpidem​ (Ambien) for treatment of​ short-term insomnia. Which is the primary safety concern of the​ nurse? A. Sleepwalking B. Dizziness and daytime sedation C. Nausea D. Diarrhea

A. Sleepwalking

A client with​ Parkinson's disease is experiencing an increase in bradykinesia. Which assessment findings should the nurse​ anticipate? Select all that apply. A. Slow speech B. Difficulty chewing C. Shuffling the feet when walking D. Stooped posture E. Lack of facial expression

A. Slow speech B. Difficulty chewing C. Shuffling the feet when walking

The nurse preparing the education for a client prescribed risperidone​ (Risperdal). For which condition should the nurse inform the client to immediately notify the healthcare​ provider? A. Sore throat or an infection. B. Joint pain or swelling. C. Yellowish halos around lights. D. Narrowing of the field of vision.

A. Sore throat or an infection.

The educator is reviewing the neural mechanisms of pain with a nurse. Which information should the educator provide about substance​ P? A. Substance P controls which pain signals reach the brain. B. Substance P modifies sensory information in the spinal cord. C. Substance P is also known as an endogenous opioid. D. Substance P stimulates pain receptors in the spinal cord.

A. Substance P controls which pain signals reach the brain.

A student nurse asks a nurse which client situations are considered involuntary responses to autonomic nervous system​ control? What is the​ nurse's best​ response(s)? Select all that apply. A. Sweating when hot. B. Stepping over a chair to prevent falling. C. Complaining of nausea. D. Salivating at the smell of food. E. Breathing deeper when running.

A. Sweating when hot. D. Salivating at the smell of food. E. Breathing deeper when running.

Which assessment findings should the nurse anticipate if a client is diagnosed with​ post-traumatic stress​ disorder? Select all that apply. A. Tachycardia B. Panic attacks C. Inability to focus D. A fear of exposure to germs E. Hallucinations

A. Tachycardia B. Panic attacks C. Inability to focus E. Hallucinations

The nurse understands that which client description is most desirable after the administration of general anesthesia prior to a surgical​ procedure? A. The client is unconscious with slow eye movements. B. The client is unresponsive to pain and is unable to breath spontaneously. C. The client loses​ sensation, but remains awake. D. The client is​ unconscious, with arm and leg movement.

A. The client is unconscious with slow eye movements.

Which assessment findings should the nurse associated with a risk for​ suicide? Select all that apply. A. The client states that​ "suicide is always an​ option." B. The client describes a previous unsuccessful attempt at suicide by aspirin overdose. C. The client states that the prescribed medication is not working and that feelings of depression are worse. D. The client requests prescriptions for pain medication and a sleeping aid. E. The client expresses interest in meeting with friends more often.

A. The client states that​ "suicide is always an​ option." B. The client describes a previous unsuccessful attempt at suicide by aspirin overdose. C. The client states that the prescribed medication is not working and that feelings of depression are worse. D. The client requests prescriptions for pain medication and a sleeping aid.

The nurse is preparing to administer medications to a group of clients. One of the medications is​ atropine, an anticholinergic drug. This drug is contraindicated in which​ client? A. The client with glaucoma. B. The client with lung cancer. C. The client with a hiatal hernia. D. The client with hyperthyroidism.

A. The client with glaucoma.

A student nurse is learning about a medication that affects the autonomic nervous system. When instructing the student about the effects of this​ medication, the nurse will begin by explaining the basic structures of this system. What does this​ include? Select all that apply. A. The preganglionic neuron. B. The postganglionic neuron. C. The synaptic cleft. D. Norepinephrine. E. Dopamine.

A. The preganglionic neuron. B. The postganglionic neuron. C. The synaptic cleft.

The nurse is preparing to educate a client on prescribed antiepileptic drug​ (AED). Which information should the nurse include in the​ teaching? Select all that apply. A. The prescription should be taken at the same time every day. B. When a dose of the prescription is missed take both doses together when the next does is due. C. Take an additional dose of the prescription if experiencing a seizure aura. D. When experiencing side effects skip the next dose of the prescription. E. Avoid the use of dietary supplements containing kava while taking the prescription.

A. The prescription should be taken at the same time every day. E. Avoid the use of dietary supplements containing kava while taking the prescription.

Which explanation best explains why barbiturates are rarely used to treat anxiety and​ insomnia? A. They produce many serious adverse effects. B. They have a high risk of producing an allergic response. C. They have a greater associated cost. D. They are seldom effective.

A. They produce many serious adverse effects.

The nurse suspects a client has received too much phenylephrine​ (Neo-Synephrine). Which findings support the​ nurse's suspicion? Select all that apply. A. The​ client's heart rate has increased from 72 bpm to 114 bpm. B. The client reports feeling anxious. C. The​ client's blood pressure has increased from​ 140/86 mmHg to​ 180/98 mmHg. D. The​ client's stools are dark and tarry. E. The client reports burning at the injection site.

A. The​ client's heart rate has increased from 72 bpm to 114 bpm. C. The​ client's blood pressure has increased from​ 140/86 mmHg to​ 180/98 mmHg.

The nurse is assessing a client with schizophrenia prescribed a phenothiazine for anticholinergic side effects. Which finding should the nurse immediately report to the healthcare​ provider? A. Urinary retention B. Acute dystonia C. Complaint of a severe headache D. Hypertension

A. Urinary retention

The nurse is providing client teaching about the side effects of amitriptyline​ (Elavil). Which information should the nurse include in the teaching to promote prescription​ adherence? Select all that apply. A. Use ice chips to help alleviate dry mouth. B. Chew gum or use hard candy to help alleviate dry mouth. C. Avoid​ alcohol-based mouthwash to help alleviate dry mouth. D. Use​ "dry eye" drops to help with eye dryness. E. Request the pharmacist to fill the prescription from a different manufacturer.

A. Use ice chips to help alleviate dry mouth. B. Chew gum or use hard candy to help alleviate dry mouth. C. Avoid​ alcohol-based mouthwash to help alleviate dry mouth. D. Use​ "dry eye" drops to help with eye dryness.

Which information should the nurse include in the teaching for a client with a prescription for​ migraines? Select all that apply. A. Use the medication at the first sign of migraine. B. Overusing this medication may result in rebound headaches. C. Instill the spray into one nostril only. D. Increase dietary intake of​ tyramine-containing foods. E. Pain relief will not begin for up to an hour after administration.

A. Use the medication at the first sign of migraine. B. Overusing this medication may result in rebound headaches. C. Instill the spray into one nostril only.

A client is prescribed an anticholinergic drug. What discharge instructions should the nurse​ provide? Select all that apply. A. Wear sunglasses in bright light. B. Limit fluid intake. C. Increase fiber intake. D. Avoid hot showers. E. Avoid milk and dairy products.

A. Wear sunglasses in bright light. C. Increase fiber intake. D. Avoid hot showers.

A client prescribed sertraline​ (Zoloft) reports to the nurse that he is experiencing delayed ejaculation. Which response should the nurse provide the​ client? A. ​ "I will let your healthcare provider know and most likely your prescription will be​ changed." B. ​"I am concerned that you may become suicidal if you stop the​ prescription." C. ​"This is a side effect of the​ prescription, but treating your depression is the​ priority." D. ​ "Continue to take the​ prescription, as this usually goes away after a few​ months."

A. ​ "I will let your healthcare provider know and most likely your prescription will be​ changed."

Which instruction should the nurse include for a client prescribed acetaminophen​ (Tylenol)? A. ​"Acetaminophen (Tylenol) can cause your mouth to become​ dry." B. ​"You may experience diarrhea while taking acetaminophen​ (Tylenol)." C. ​"Check your gums for bleeding when taking acetaminophen​ (Tylenol)." D. ​"Do not take any narcotics with acetaminophen​ (Tylenol)."

A. ​"Acetaminophen (Tylenol) can cause your mouth to become​ dry."

The nurse is preparing education for a client that will receive anesthesia for a surgical procedure. Which statement should the nurse include in the​ teaching? A. ​"An intravenous agent will initially administer to induce sleep followed by an inhaled​ agent." B. ​"If the inhaled agent is​ ineffective, an intravenous agent will be​ administered." C. ​"Usually all that is required is an intravenous agent to induce​ sleep." D. ​"An inhaled agent is used to induce​ sleep, followed by an intravenous agent for​ relaxation."

A. ​"An intravenous agent will initially administer to induce sleep followed by an inhaled​ agent."

A​ 48-year-old male client tells the nurse they want to stop taking their prazosin​ (Minipress). Which questions should the nurse ask the​ client? Select all that apply. A. ​"Are you experiencing​ impotence?" B. ​"Are you experiencing nasal​ congestion?" C. ​"Do you feel​ tired?" D. ​"Do you feel​ nervousness?" E. ​"Do you feel like you heart rates slows​ down?"

A. ​"Are you experiencing​ impotence?" B. ​"Are you experiencing nasal​ congestion?" C. ​"Do you feel​ tired?"

The nurse has provided education for a patient with depression. Which statement made by the patient indicates an understanding of the​ teaching? Select all that apply. A. ​"Depression has many causes which include​ brain-based disorders." B. ​"Depression can be an inherited​ illness." C. ​"We really​ don't know what causes​ depression." D. ​"Depression results from unresolved conflicts in your​ childhood." E. ​"Depression results from parental​ neglect."

A. ​"Depression has many causes which include​ brain-based disorders." B. ​"Depression can be an inherited​ illness."

An older adult has received adrenergic eye drops prior to a retinal exam. Which information should the nurse include in the​ client's discharge​ teaching? Select all that apply. A. ​"Do not drive until the effects of the eye drops have worn​ off." B. ​"Wear sunglasses when in bright​ light." C. ​"You may be more comfortable in a darkened​ room." D. ​"You may experience burning in your eyes for a couple of​ days." E. ​"Do not eat or drink anything for at least an hour after​ discharge."

A. ​"Do not drive until the effects of the eye drops have worn​ off." B. ​"Wear sunglasses when in bright​ light." C. ​"You may be more comfortable in a darkened​ room."

A client has been prescribed an adrenergic nasal spray. Which information should the nurse include in the​ teaching? Select all that apply. A. ​"Do not share this spray with​ anyone." B. ​"Sit upright while using this​ spray." C. ​"Only use this spray for 3-5 ​days." D. ​"Do not shake the bottle before using this​ spray." E. ​"Keep this spray​ refrigerated."

A. ​"Do not share this spray with​ anyone." B. ​"Sit upright while using this​ spray." C. ​"Only use this spray for 3-5 ​days."

The nurse is preparing to provide education about the nonpharmacological treatment of muscle spasms for a client with a neuromuscular disorder. Which information should the nurse​ include? Select all that apply... A. Application of heat or cold B. Ultrasound C. Massage D. Relaxation techniques E. Guided imagery

A. B. and C.

The nurse is admitting a client that hears voices telling them to harm them self. Which questions should the nurse ask the​ client? Select all that apply. A. ​"Do you have a family history of​ first-degree relative with​ schizophrenia?" B. ​"Do you use any street​ drugs?" C. ​"Have you recently experienced any episodes of​ anxiety?" D. ​"Do you have a family history of heart​ disease?" E. ​"Have you experienced any emotional or physical​ abuse?"

A. ​"Do you have a family history of​ first-degree relative with​ schizophrenia?" B. ​"Do you use any street​ drugs?" E. ​"Have you experienced any emotional or physical​ abuse?"

A parent of a child pharmacologically treated for ADHD asks the nurse to explain a​ "drug holiday." Which information should the nurse provide about the treatment​ option? Select all that apply. A. ​"Drug holidays may be used to reduce dependence on medications or to assess​ symptoms." B. ​"If you feel like the medication is sedating your child too​ much, you can hold doses for up to 3 or 4​ days." C. ​"Drug holidays are used when the dosage will be increased to assess the​ patient's tolerance to the​ prescription." D. ​"You should not consider using a drug holiday until you have discussed that option with your healthcare​ provider." E. ​"Drug holidays require careful​ scheduling."

A. ​"Drug holidays may be used to reduce dependence on medications or to assess​ symptoms." D. ​"You should not consider using a drug holiday until you have discussed that option with your healthcare​ provider." E. ​"Drug holidays require careful​ scheduling."

A client with schizophrenia whose symptoms have been controlled with a liquid prescription has reported both positive and negative symptoms have returned over the last week. Which questions should the nurse ask the​ patient? Select all that apply. A. ​"Have you mixed your prescription in any​ liquids?" B. ​"Have you changed​ pharmacies?" C. ​"What was your last fasting blood​ glucose?" D. ​"Have you the started smoking​ again?" E. ​"Are you consuming more dairy products than​ usual?"

A. ​"Have you mixed your prescription in any​ liquids?" B. ​"Have you changed​ pharmacies?" D. ​"Have you the started smoking​ again?"

A client recently prescribed an adrenergic drug states to the​ nurse, "I am so nervous and I cannot​ sleep." Based on the​ client's statement, which questions should the nurse​ ask? Select all that apply. A. ​"How much coffee do you​ drink?" B. ​"Have you recently increased your intake of dairy​ products?" C. ​"Do you eat​ chocolate?" D. ​"When was the last time you ate pickled foods or aged​ cheese?" E. ​"How much wine or other alcoholic beverages do you​ drink?"

A. ​"How much coffee do you​ drink?" C. ​"Do you eat​ chocolate?"

The nurse is preparing to assess an older adult for depression. Which questions should the nurse include in the​ assessment? Select all that apply. A. ​"How often do you go out to socialize with​ friends?" B. ​"Can you tell me about your​ self-care activities." C. ​"How much alcohol do you consume every​ day?" D. ​"How is living with your oldest daughter and her family working out for​ you?" E. ​"Are you feeling​ depressed?"

A. ​"How often do you go out to socialize with​ friends?" B. ​"Can you tell me about your​ self-care activities." C. ​"How much alcohol do you consume every​ day?" D. ​"How is living with your oldest daughter and her family working out for​ you?"

Which comment by the client receiving morphine should the nurse recognize as an adverse​ effect? A. ​"I feel like I am going to throw​ up." B. ​"My ears are constantly​ ringing." C. ​"My heart feels like it is skipping​ beats." D. ​"I feel cold shivers all​ over."

A. ​"I feel like I am going to throw​ up."

The nurse has provided education for a client who received a new prescription for the treatment of​ Parkinson's disease. Which statements made by the client indicate an understanding of the​ information? Select all that apply. A. ​"I guess I will have to give up eating my​ wife's banana​ pudding." B. ​"I should add some wheat germ to my breakfast​ cereal." C. ​"I should add olive oil to the salads I eat at noon each​ day." D. ​"I will most likely have to become a​ vegetarian." E. ​"I will take my medication on an empty​ stomach."

A. ​"I guess I will have to give up eating my​ wife's banana​ pudding." E. ​"I will take my medication on an empty​ stomach."

The nurse has completed the education for a client prescribed levodopa​ (Stalevo). Which statement made by the client indicates further education is​ needed? A. ​"I will weigh myself​ weekly." B. ​"I will increase the intake of vitamin B6 in my​ diet." C. ​"I will notify my healthcare provider if my urine​ darkens." D. ​"I will increase the protein in my​ diet."

A. ​"I will weigh myself​ weekly."

A client has been prescribed an oral drug containing atropine. The nurse would hold the drug and contact the prescriber if the client states which of the​ following? Select all that apply. A. ​"I would like to wait a few minutes to take this as I just drank some aloe​ juice." B. ​"Can I take this at the same time as my​ procainamide?" C. ​"I have had a headache this​ morning." D. ​"I am allergic to​ penicillin." E. ​"My gallbladder surgery is scheduled for next​ week."

A. ​"I would like to wait a few minutes to take this as I just drank some aloe​ juice." B. ​"Can I take this at the same time as my​ procainamide?"

The educator has discussed the protocol for the immediate availability of naloxone for a client with a​ patient-controlled analgesia PCA pump with a nurse. Which statement made by the nurse indicates an understanding of the​ information? A. ​"Naloxone ​(Narcan) will reverse the effects of the narcotic in the​ patient-controlled analgesia​ (PCA) pump if an overdose​ occurs." B. ​"Naloxone ​(Narcan) enhances the effect of the opioid in the​ patient-controlled analgesia​ (PCA) pump and increases​ analgesia." C. ​"Naloxone ​(Narcan) is the antidote if an anaphylactic reaction to the opioid in the​ patient-controlled analgesia​ (PCA) pump​ occurs." D. ​"Naloxone ​(Narcan) is available to treat any systemic side​ effects, like​ constipation, of the opioid in the​ patient-controlled analgesia​ (PCA) pump."

A. ​"Naloxone ​(Narcan) will reverse the effects of the narcotic in the​ patient-controlled analgesia​ (PCA) pump if an overdose​ occurs."

The nurse has provided discharge teaching for a client who had an abobotulinumtoxinA​ (Dysport) injection. Which symptoms should the nurse instruct the client to immediately​ report? Select all that apply... A. Chest pain B. Urinary retention C. Heart palpitations D. Difficulty swallowing E. Eye spasm

A. B. and D

The nurse has provided client education about the use of nonpharmacological techniques for pain management. Which statements indicate the client understands the​ information? Select all that apply. A. ​"Nonpharmacological techniques are a good adjunct to​ pharmacotherapy." B. ​"Nonpharmacological techniques have not reached mainstream​ yet." C. ​"Nonpharmacological techniques may be used in place of​ drugs." D. ​"Nonpharmacological techniques include an aerobic​ exercise." E. ​"Nonpharmacological techniques are not usually valued by​ nurses."

A. ​"Nonpharmacological techniques are a good adjunct to​ pharmacotherapy." C. ​"Nonpharmacological techniques may be used in place of​ drugs."

The educator has discussed the types of opioid receptors with a nurse. Which statement made by the nurse indicates an understanding of the​ information? A. ​"Opioid agonists will activate mu and kappa​ receptors, producing an analgesic​ effect." B. ​"The sigma and kappa receptors have been traditionally targeted for pain​ management." C. ​"Drugs that block opioid receptors inhibit the pain​ impulse." D. ​"Opioids exert their actions by interacting with three different​ receptors."

A. ​"Opioid agonists will activate mu and kappa​ receptors, producing an analgesic​ effect."

A client that has been taking phenytoin​ (Dilantin) tells the nurse they want to stop taking the medication due to the side effects. Which response should the nurse provide the​ client? A. ​"Please do not stop the prescription​ abruptly, as you will have withdrawal​ seizures." B. ​"Another prescription can be added to decrease the side​ effects." C. ​"Your seizures have been controlled so I will let your healthcare provider know that you no longer will be taking the​ prescription." D. ​"Side effects are a​ problem, but they are not as bad as the seizures you were​ having."

A. ​"Please do not stop the prescription​ abruptly, as you will have withdrawal​ seizures."

The nurse has completed group education for clients with anxiety disorders. Which statement made be a client indicates an understanding of the​ education? Select all that apply. A. ​"Relaxation techniques will often decrease​ anxiety." B. ​"Antianxiety medicine should be used until anxiety is​ gone." C. ​"Antianxiety medicine should not be used​ indefinitely." D. ​"Therapy may help identify where this anxiety comes​ from." E. ​"Different medications are required for treating anxiety than for treating difficulty​ sleeping."

A. ​"Relaxation techniques will often decrease​ anxiety." C. ​"Antianxiety medicine should not be used​ indefinitely." D. ​"Therapy may help identify where this anxiety comes​ from."

A client has been prescribed an​ adrenergic-blocker for treatment of hypertension. Which information should the nurse​ provide? Select all that apply. A. ​"Rise from a sitting position slowly B. ​"Sit on the side of the bed a few moments before you​ stand." C. ​"If you feel​ dizzy, add more salt to your​ diet." D. ​"If you feel​ dizzy, sit or lie down until the dizziness​ passes." E. ​"Dizziness is expected so just continue your daily​ activities."

A. ​"Rise from a sitting position slowly B. ​"Sit on the side of the bed a few moments before you​ stand." D. ​"If you feel​ dizzy, sit or lie down until the dizziness​ passes."

The nurse has completed the education for the parents of a child with​ tonic-clonic seizures. Which statements made by the parents indicate further education is​ needed? Select all that apply. A. ​"Some of the times when I thought he was ignoring me may have actually been seizure​ activity." B. ​"He just needs to focus more to prevent these​ attacks." C. ​"I know he will outgrow these seizures with​ time." D. ​"I hope we can help our son identify his seizure​ aura." E. ​"We will watch for the development of status​ epilepticus."

A. ​"Some of the times when I thought he was ignoring me may have actually been seizure​ activity." B. ​"He just needs to focus more to prevent these​ attacks." C. ​"I know he will outgrow these seizures with​ time."

The nursing instructor teaches the student nurses about the nervous system. The instructor determines that learning has occurred when the students make which​ statement(s)? Select all that apply. A. ​"The central nervous system includes the brain and spinal​ cord." B. ​"The peripheral nervous system has mainly sensory​ functions." C. ​"The somatic nervous system gives us voluntary control over our gastrointestinal​ (GI) tract." D. ​"The nervous system helps us react to environmental​ changes." E. ​"The somatic nervous system gives us voluntary control over​ moving."

A. ​"The central nervous system includes the brain and spinal​ cord." D. ​"The nervous system helps us react to environmental​ changes." E. ​"The somatic nervous system gives us voluntary control over​ moving."

A parent​ states, "The doctors all say that my​ son's problem is​ epilepsy, but I​ don't think that is correct. I have never seen him jerk or​ thrash." Which information should the nurse include in a response to the​ parent? Select all that apply. A. ​"The episodes of jerking and thrashing are called​ convulsions." B. ​"Seizures take many​ forms." C. ​"Your son may have​ seizures, but not​ epilepsy." D. ​"Seizures can be a sign that epilepsy is​ present." E. ​"If there is no physical​ movement, epilepsy is ruled​ out."

A. ​"The episodes of jerking and thrashing are called​ convulsions." B. ​"Seizures take many​ forms." D. ​"Seizures can be a sign that epilepsy is​ present."

A client ask the nurse which part of the brain controls anxiety and insomnia. Which response should the nurse​ provide? A. ​"The limbic system and reticular activating system control anxiety and​ insomnia." B. ​"The limbic system and hypothalamus control anxiety and​ insomnia." C. ​"The thalamus and reticular activating system control anxiety and​ insomnia." D. ​"The frontal lobes and limbic system control anxiety and​ insomnia."

A. ​"The limbic system and reticular activating system control anxiety and​ insomnia."

A client with​ Parkinson's disease asks the nurse how his or her prescriptions will help him or her. Which response should the nurse​ provide? A. ​"The prescriptions will help you to eat and​ walk." B. ​"The prescriptions will increase your appetite and​ energy." C. ​"The prescriptions will help prevent muscle​ wasting." D. ​"The prescriptions will balance serotonin and acetylcholine in​ client's brain."

A. ​"The prescriptions will help you to eat and​ walk."

Which drug toxicity is a client experiencing that exhibits tachycardia and​ palpitations? A. Benzodiazepines B. Nicotine C. Marijuana D. Alcohol

B

Which statements made by the client indicate he or she may be experiencing​ psychosis? Select all that apply. A. ​"The servants need to be​ disciplined." B. ​"The voice told me to cut​ myself." C. ​"I have not had an appetite for​ weeks." D. ​"Close the door so that man​ can't get to​ me." E. ​"I have not been able to get any sleep for​ months."

A. ​"The servants need to be​ disciplined." B. ​"The voice told me to cut​ myself." D. ​"Close the door so that man​ can't get to​ me."

The nursing instructor teaches the student nurses about the autonomic nervous system. The instructor determines that learning has occurred when the students make which​ statement(s)? Select all that apply. A. ​"The sympathetic and parasympathetic systems are not always opposite in their​ effects." B. ​"The parasympathetic nervous system is the​ "fight-or-flight" response." C. ​"Sympathetic stimulation causes dilation of​ arterioles." D. ​"The parasympathetic nervous system causes bronchial​ constriction." E. ​"The sympathetic nervous system is activated under​ stress."

A. ​"The sympathetic and parasympathetic systems are not always opposite in their​ effects." D. ​"The parasympathetic nervous system causes bronchial​ constriction." E. ​"The sympathetic nervous system is activated under​ stress."

A client has been prescribed phenylephrine​ (Neo-Synephrine) spray for nasal congestion. Which information about adverse effects should the nurse provide in discharge​ teaching? Select all that apply. A. ​"This drug may cause some stinging or burning in your​ nose." B. ​"You may notice that your nasal secretions take on a slightly orange​ tint." C. ​"You may feel like your blood pressure is low for the first few times you use this​ spray." D. ​"Rebound congestion may occur if the prescription is used more than a few​ days." E. ​"Do not drink herbal teas while taking this​ medication."

A. ​"This drug may cause some stinging or burning in your​ nose." D. ​"Rebound congestion may occur if the prescription is used more than a few​ days."

The nurse has provided education for a client prescribed a topical anesthetic. Which statements made by the client indicate an understanding of the​ information? Select all that apply. A. ​"This lotion should only be used on small areas of​ skin." B. ​"I will wear gloves when I apply the​ lotion." C. ​"This lotion will work well on cuts​ too." D. ​"I will wash my hands before touching my​ eyes." E. ​"I will be mindful of the expiration date on the​ prescription."

A. ​"This lotion should only be used on small areas of​ skin." D. ​"I will wash my hands before touching my​ eyes." E. ​"I will be mindful of the expiration date on the​ prescription."

A client asks the nurse why he or she are prescribed different types of medications for his or her anxiety and insomnia. Which response should the nurse provide the​ client? Select all that apply. A. ​"You will be taking medications known as​ sedative-hypnotics." B. ​"You will be taking medications known as​ antidepressants." C. ​"You will be taking a medication known as​ paraldehyde." D. ​"You will be taking medications known as​ barbiturates." E. ​"You will be taking medications known as​ benzodiazepines."

A. ​"You will be taking medications known as​ sedative-hypnotics." B. ​"You will be taking medications known as​ antidepressants." E. ​"You will be taking medications known as​ benzodiazepines."

A nurse is providing teaching to a client who has a prescription for buspirone to treat anxiety. Which of the following instructions should the nurse include? A. "Take a missed dose as soon as you remember." B. "Take the drug as needed at the first sign of anxious feelings." C. "Take the drug in the morning with a glass of grapefruit juice." D. "Drink a cup of chamomile tea at night with the drug."

A. "Take a missed dose as soon as you remember."

A nurse is assessing a client who has been taking alprazolam for several days. Which of the following findings should the nurse identify as an adverse reaction caused by alprazolam? A. Anxiety B. Ringing in the ears C. Increased appetite D. Muscle spasms

A. Anxiety

A nurse should identify that timolol is contraindicated for a client who has which of the following disorders? A. Asthma B. Seizure disorder C. Diabetes mellitus D. Rheumatoid arthritis

A. Asthma

Which statement made by a client prescribed cyclobenzaprine​ (Amrix) requires additional​ teaching? Select all that apply... A. ​"My wife and I are driving our motor home to the beach​ tomorrow." B. ​"I am going home to mow my​ lawn." C. ​"I take a nap every​ afternoon." D. ​"I just checked out a new book from the​ library." E. ​"I am taking my grandchildren to the zoo​ tomorrow."

A. B. and E.

A nurse is reviewing the drug history of a client who is taking lithium carbonate for bipolar disorder. Which of the following findings should indicate to the nurse to monitor for lithium toxicity? A. Furosemide for hypertension B. Acetaminophen for headaches C. Ciprofloxacin for a urinary tract infection D. Montelukast for asthma

A. Furosemide for hypertension

A nurse is caring for a client who has a prescription for fluoxetine and who reports self-administering St. John's wort daily for the past 2 weeks. Which of the following findings should the nurse report to the provider as an indication of serotonin syndrome? A. Hallucinations B. Decreased temperature C. Hypersexual behavior D. Constipation

A. Hallucinations

A nurse is reviewing the medical history of a client who has a new prescription for a reduced dosage of alprazolam for anxiety. Which of the following findings should indicate to the nurse the reason for the prescription modification? A. Renal function impairment B. Cataracts C. Recent weight loss D. Smoking

A. Renal function impairment

A client that experienced malignant hyperthermia is being treated with dantrolene. Which assessment findings should the nurse recognize are an adverse reaction to the​ treatment? Select all that apply... A. Swelling at the IV site B. Muscle weakness C. Sore throat D. Irritability E. Anorexia

A. and B.

Which information should the nurse include in the education of the body movement for a client with a neuromuscular​ disorder? Select all that apply... A. Body movement depends on proper functioning of muscles. B. Body movement depends on intact nerves. C. Body movement depends on an intact spinal cord. D. Body movement depends on proper endocrine functioning.

A. and B.

The nurse is reviewing the records of a client prescribed cyclobenzaprine​ (Amrix). Which findings should the nurse be most concerned​ about? Select all that apply... A. The client reports drinking wine each day with dinner. B. The client has a history of using nasal saline spray for allergies. C. The client was prescribed a MAO inhibitor last month. D. The​ client's blood glucose was elevated at the last visit. E. The client had an ulnar fracture repaired a year ago.

A. and C.

Which clients prescribed dantrolene​ (Dantrium) should the nurse recognize have the greatest risk for liver​ toxicity? Select all that apply... A. A​ 49-year-old woman who has a history of esophagitis. B. A​ 60-year-old man with pneumonia. C. A​ 38-year-old woman who has type 2 diabetes. D. An​ 18-year-old who injured his leg playing soccer. E. A​ 26-year-old woman who has an ostomy.

A. and C.

A nurse is teaching a client who has a new prescription for allopurinol. Which of the following instructions should the nurse include? -Avoid driving or activities that require mental alertness. -avoid crushing the tablets -limit fluid intake during therapy -limit potassium while taking allopurinol

Avoid driving or activities that require mental alertness.

A client receiving a pain prescription for several weeks has required an increase of the dosage to adequately manage symptoms. Which should the nurse recognize has​ occurred? A. Immunity B. Tolerance C. Physiologic addiction D. Psychological addiction

B

A client tells the nurse he or she would like to stop taking methamphetamines but cannot stop. Which should the nurse recognize the client is​ experiencing? A. Physical dependence B. Substance dependence C. Psychological dependence D. Withdrawal syndrome

B

The nurse has provided education to a client about abuse of methylphenidate. Which response by the client indicates further teaching is​ required? A. ​"The prescription gives them more​ energy." B. ​"The prescription produces​ relaxation." C. ​"The prescription produces a​ euphoria." D. ​"The prescription heightens the​ awareness."

B

The nurse assessing a client suspected of hallucinogen toxicity. Which assessment findings support the​ nurse's suspicion? Select all that apply. A. Somnolence B. Confusion C. Panicked reactions D. Blurred vision E. Respiratory depression

B,C,D

The nurse has provided group education on the topic of addiction. Which statements made by the clients indicate an understanding of the​ information? Select all that apply. A. ​"Heroin addicts often die from heroin​ withdrawal." B. ​"Substance abuse depends on complex​ variables." C. ​"Most addicts became addicted from pain medication in a​ hospital." D. ​"Addiction includes a compulsion to use a​ mood-altering substance." E. ​"There is most likely a genetic component to​ addiction."

B,D,E

The educator is reviewing botulinum toxin type B​ (Myobloc) with a nurse. Which should the educator include in the​ discussion? A. The therapeutic effect can last up to a year. B. High doses are toxic causing food poisoning. C. It can take 6 months to experience the therapeutic effect. D. Classified as a cholinergic agonist.

B.

The nurse is preparing to provide education for a client prescribed cyclobenzaprine​ (Amrix). Which information should the nurse include in the​ education? A. Restrict the intake of sodium. B. Increase the intake of fiber. C. Increase the intake of protein. D. Avoid the use of caffeine.

B.

The nurse suspects that a client is developing malignant hyperthermia. Which is the priority nursing​ intervention? A. Control associated symptoms. B. Rapidly assess for supporting signs and symptoms. C. Discontinue the triggering prescription. D. Administer dantrolene intravenously.

B.

Which area of the body when damaged commonly results in​ spasticity? A. Brainstem B. Cerebral cortex C. Spinal cord D. Peripheral nerves

B.

Which lab result is a priority for the nurse to assess for a client who is prescribed dantrolene​ (Dantrium)? A. Hemoglobin and hematocrit. B. Aspartate aminotransferase​ (AST) and alanine aminotransferase​ (ALT). C. Serum amylase. D. Creatinine clearance.

B.

A client who has history of severe arteriosclerosis requires a repair of a laceration. Which property contained in a local anesthetic should the nurse recognize is safe to administer to the​ client? A. Epinephrine B. Sodium bicarbonate C. Proparacaine​ (Alcaine) D. Benzocaine

B. Sodium bicarbonate

Which statement describes the primary difference between centrally acting muscle relaxants and​ direct-acting antispasmodics? A. Centrally acting agents stimulate the central nervous​ system, while​ direct-acting agents inhibit neuronal conduction of the central nervous system. B. Centrally acting agents inhibit neurons of the central nervous​ system, while​ direct-acting agents work at the level of the neuromuscular junction and skeletal muscles. C. Centrally acting agents stimulate neurons of the central nervous​ system, while​ direct-acting agents stimulate neurons of the peripheral nervous system. D. Centrally acting agents inhibit neurons of the central nervous​ system, while​ direct-acting agents stimulate neurons of the central nervous system.

B.

A nursing instructor is planning to teach nursing students about exogenous acetylcholine. The instructor should include which​ information? A. Acetylcholine will cause the heart rate to increase and blood pressure to drop. B. Acetylcholine has almost no therapeutic effects because it is rapidly destroyed once given. C. When given in small​ amounts, acetylcholine will produce profound parasympathetic effects. D. Acetylcholine is broken down rapidly within the​ body, preventing it from producing adverse effects.

B. Acetylcholine has almost no therapeutic effects because it is rapidly destroyed once given.

A client overdosed on risperidone​ (Risperdal). Which intervention should the nurse be prepared to​ perform? A. Provide symptom management B. Administer activated charcoal C. Administer a CNS depressant D. Administer anticholinergic

B. Administer activated charcoal

Which statement regarding the use of zolpidem​ (Ambien) for insomnia is​ accurate? A. Ambien is classified as a benzodiazepine. B. Ambien will take longer to produce an effect when taken with food. C. Clients using Ambien should avoid foods that contain tyramine. D. Ambien is contraindicated during pregnancy but can be taken by breastfeeding mothers.

B. Ambien will take longer to produce an effect when taken with food.

Which client conditions should the nurse anticipate to be treated with a beta2​ agonists? Select all that apply. A. Heart failure B. Asthma C. COPD D. Overactive bladder E. Nasal congestion

B. Asthma C. COPD

For which client receiving an inhalation anesthesia should the nurse be most​ concerned? A. A​ 55-year-old with a serious neck injury B. A​ 79-year-old with arteriosclerosis C. A​ 15-year-old with diabetes mellitus D. A​ 6-year-old with no chronic health problems

B. A​ 79-year-old with arteriosclerosis

For which adverse effect should the nurse monitor a client prescribed phenytoin​ (Dilantin)? A. Hypoglycemia B. Bleeding C. Vitamin B deficiency D. Leg edema

B. Bleeding

A client has been prescribed scopolamine​ (Transderm-Scop) for the prevention of motion sickness. The nurse should teach the client to immediately report which adverse​ effects? Select all that apply. A. Slow heart rate B. Cardiac palpitations C. Decreased urinary output D. Development of tremors E. Diarrhea

B. Cardiac palpitations C. Decreased urinary output D. Development of tremors

Which atypical antipsychotic should the nurse anticipate a client with schizophrenia demonstrating both positive and negative symptoms will be​ prescribed? A. Chlorpromazine​ (Thorazine) B. Clozapine​ (Clozaril) C. Thioridazine HCL​ (Mellaril) D. Haloperidol​ (Haldol)

B. Clozapine​ (Clozaril)

The nurse is preparing education for a caregiver of a patient with​ Alzheimer's disease. Which information should the nurse include in the​ teaching? Select all that apply. A. Glutamergic inhibitors are the most common class of drugs for treating​ Alzheimer's disease. B. Depression and aggressive behavior are common with the disease. C. Memory difficulties are an early symptom of the disease. D. Chronic inflammation of the brain can be a causative actor in the condition. E. Pharmacologic therapies are given to help improve memory in​ Alzheimer's disease.

B. Depression and aggressive behavior are common with the disease. C. Memory difficulties are an early symptom of the disease. D. Chronic inflammation of the brain can be a causative actor in the condition. E. Pharmacologic therapies are given to help improve memory in​ Alzheimer's disease.

A client with a preexisting conditions states to the​ nurse, "I took my blood pressure at home and it was high so I have been taking my​ husband's propranolol​ (Inderal) for the last​ week." Based on the​ client's statement, which preexisting conditions should the nurse be most concerned​ with? Select all that apply. A. Obesity B. Diabetes mellitus C. COPD D. Asthma E. Rheumatoid arthritis

B. Diabetes mellitus C. COPD D. Asthma

The nurse is reviewing the prescription history for a client scheduled for an operative procedure that will require the administration of succinylcholine​ (Anectine). Which prescription should the nurse find most​ concerning? A. Citalopram B. Diazepam C. St.​ John's Wort D. Furosemide

B. Diazepam

The nurse is preparing to administer an adrenergic drug intravenously. Which should the nurse include in the plan of​ care? Select all that apply. A. Give the drug as rapidly as possible by intravenous push. B. Dilute the drug before administration. C. Use an infusion pump to control rate of administration. D. Monitor for blanching at the infusion site. E. Advise the patient that a sweet taste may occur as the drug is being given.

B. Dilute the drug before administration. C. Use an infusion pump to control rate of administration. D. Monitor for blanching at the infusion site.

Which anatomical areas should be avoided when administering lidocaine with​ epinephrine? ​Note: Credit will be given only if all correct choices and no incorrect choices are selected. Select all that apply. A. Cheek B. Earlobe C. Tip of the nose D. Top of the foot E. Fingertip

B. Earlobe C. Tip of the nose E. Fingertip

A student nurse asks a nursing instructor which part of the nervous system primarily regulates the ability of a person to use his arm muscles to lift a weight. What is the​ instructor's best​ response? A. Parasympathetic nervous system. B. Somatic nervous system. C. Sympathetic nervous system. D. Autonomic nervous system.

B. Somatic nervous system.

The nurse is preparing the education for a client with progressive multiple sclerosis prescribed alemtuzumab​ (Lemtrada). Which information should the nurse plan to include in the​ education? Select all that apply. A. Monitoring for hypertension B. Evaluation of a complete blood count​ (CBC) C. Requirement of two doses of treatment 12 months apart D. Dietary decrease of protein and vitamin B6 E. Intravenous administration of prescription

B. Evaluation of a complete blood count​ (CBC) C. Requirement of two doses of treatment 12 months apart E. Intravenous administration of prescription

Which barriers make it difficult to treat a client with schizophrenia demonstrating negative​ symptoms? Select all that apply. A. The client feels that their behavior is normal. B. Family members feel the client is going through a period of depression. C. The client is suspicious of the motives of the healthcare provider. D. Family members have labeled the patient as lazy. E. The client is indifferent to obtaining help.

B. Family members feel the client is going through a period of depression. D. Family members have labeled the patient as lazy. E. The client is indifferent to obtaining help.

The​ health-care provider has ordered bethanechol​ (Urecholine), a cholinergic​ drug, for the client with urinary retention. The client also has an enlarged prostate gland. What is the priority action by the​ nurse? A. Administer the drug and push fluids. B. Hold the drug and contact the​ health-care provider. C. Administer the drug and measure urinary output. D. Hold the drug and prepare to catheterize the client.

B. Hold the drug and contact the​ health-care provider.

A client who has been taking lorazepam​ (Ativan) for 2 years presents to the clinic and tells the nurse that her or she stopped taking his or her medication 3 days prior. Which assessment findings should the nurse​ anticipate? A. A sense of calmness and lack of anxiety. B. Increased heart​ rate, fever, and muscle cramps. C. Nothing​ different; it is safe to abruptly stop lorazepam​ (Ativan.) D. Pinpoint​ pupils, constipation, and urinary retention.

B. Increased heart​ rate, fever, and muscle cramps.

A client is prescribed prazosin​ (Minipress). Which information should the nurse include in the client​ education? A. Instruct the client to decrease his intake of sodium. B. Instruct the client to not take OTC herbal preparations containing saw palmetto. C. Instruct the client to wear sunglasses when outdoors D. Instruct the client to not take tub baths.

B. Instruct the client to not take OTC herbal preparations containing saw palmetto.

Which local anesthetic agent should the nurse recognize has the lowest rate of adverse​ effects? A. Chloroprocaine​ (Nesacaine) B. Lidocaine​ (Xylocaine) C. Procaine​ (Novocain) D. Tetracaine​ (Pontocaine)

B. Lidocaine​ (Xylocaine)

Which assessment finding in an older adult prescribed levodopa​ (Stalevo) should the nurse be most concerned​ with? A. Diarrhea B. Muscle twitching C. Hypertension D. Dark urine

B. Muscle twitching

A client who has been treated with an antipsychotic agent develops an elevated blood​ pressure, dyspnea, and an elevated temperature. Which disorder should the nurse suspect has​ occurred? A. Akathisia B. Neuroleptic malignant syndrome C. Extrapyramidal side effects D. Acute dystonia

B. Neuroleptic malignant syndrome

The nurse is assessing a female client who has been taking chlorpromazine. Which should the nurse include in the​ assessment? Select all that apply. A. Ask the client if she is taking any herbal medicine. B. Obtain a lipid profile. C. Ask the client questions regarding amount of alcohol intake. D. Determine the date of her last menstrual period. E. Assess thyroid function with laboratory work.

B. Obtain a lipid profile. C. Ask the client questions regarding amount of alcohol intake. D. Determine the date of her last menstrual period.

The educator is discussing the adverse effects of prescriptions used to treat seizure disorders. Which prescription should educator refer to when discussing the risk for​ extravasation? A. Ethosuximide​ (Zarontin) B. Phenytoin​ (Dilantin) C. Clonazepam​ (Klonopin) D. Phenobarbital​ (Luminal)

B. Phenytoin​ (Dilantin)

A client with a psychiatric history is attempting to communicate with the nurse using​ made-up words. Which condition is the client most likely​ exhibiting? A. Symptom of depression. B. Positive symptom of schizophrenia. C. Symptom of bipolar disorder. D. Negative symptom of schizophrenia.

B. Positive symptom of schizophrenia.

Which best describes the action of Glatiramer​ (Copaxone)? A. Stimulates the production of neurons B. Reduces the creation of new brain lesions C. Inhibits the destruction of myelin D. Stimulates the growth of myelin

B. Reduces the creation of new brain lesions

Which information should the nurse include in the teaching for a client prescribed​ ramelteon? A. Day time drowsiness is common. B. Relatively short onset of action. C. There is an increased risk of physical dependence. D. The primary action is to induce a deep sleep state.

B. Relatively short onset of action.

A client has been in the intensive care unit for a week receiving various procedures throughout the day and night. Currently the​ client, though physiologically​ stable, is irritable and paranoid and complains of vivid dreams when dozing off to sleep. Which are the best actions for the nurse to take at this​ time? Select all that apply. A. Check the​ client's oxygen status. B. Request a prescription for a for sleep hypnotic. C. Assess the​ client's vital signs. D. Turn down the lights at night and reduce noise to a minimum. E. Schedule all tests and procedures before 9 p.m. or after 7 a.m.

B. Request a prescription for a for sleep hypnotic. D. Turn down the lights at night and reduce noise to a minimum. E. Schedule all tests and procedures before 9 p.m. or after 7 a.m.

Which area of the brain is primarily responsible for maintaining sleep and​ wakefulness? A. Cerebellum B. Reticular activating system C. Cerebral cortex D. Limbic system

B. Reticular activating system

The nurse is preparing to assess a client that had a​ tonic-clonic seizure. Which assessment findings should the nurse​ anticipate? Select all that apply. A. Incontinence of urine. B. The client is in a deep sleep. C. Confusion in relation to place and time. D. The client reports of an odd taste in the mouth. E. The client attempting to remove their hospital gown.

B. The client is in a deep sleep. C. Confusion in relation to place and time.

Which is a priority outcome for a client with schizophrenia that has been prescribed haloperidol​ (Haldol)? A. The client will report symptoms of restlessness. B. The client will be compliant with taking the medication as prescribed. C. The client will consume adequate fluids and a​ high-fiber diet. D. The client will report a decrease in auditory hallucinations.

B. The client will be compliant with taking the medication as prescribed.

The nurse is preparing to administer benztropine​ (Cogentin), which is an anticholinergic drug. The nurse understands this drug is contraindicated in which​ client? A. The client with diarrhea. B. The client with tachycardia. C. The client with a fractured femur. D. The client with an irritable colon.

B. The client with tachycardia.

Which substance is used in the initial production of​ catecholamines? A. Norepinephrine B. Tyrosine C. Dopamine D. ​L-dopa

B. Tyrosine

The healthcare provider has prescribed intravenous phenytoin​ (Dilantin) IV piggyback. The client is currently receiving​ 5% dextrose in water​ (D5W) intravenously​ (IV). Which action should the nurse​ take? Select all that apply. A. Use a large vein for the infusion. B. Use an intravenous​ (IV) line with a filter. C. Flush the intravenous​ (IV) line with saline. D. Monitor the patient for hypertension. E. Monitor the patient for​ Stevens-Johnson syndrome.

B. Use an intravenous​ (IV) line with a filter. C. Flush the intravenous​ (IV) line with saline.

A client refuses his or her antipsychotic and tells the nurse he or she frequently experience a dry mouth. Which response should the nurse​ provide? A. ​"Can you tell me why you are refusing the​ medicine?" B. ​"A dry mouth is​ common, but drinking more water will​ help." C. ​"I can give you another prescription to help with the dry​ mouth." D. ​"I will let your healthcare provider know about your dry​ mouth."

B. ​"A dry mouth is​ common, but drinking more water will​ help."

Which statement should the educator use to describe to the nurse the neural mechanism responsible for signaling sharp​ pain? A. ​"Unmyelinated fibers carry the pain signal faster resulting in the feeling of sharp​ pain." B. ​"Alpha fibers are wrapped in myelin that speed up the signal resulting in the sensation of sharp​ pain." C. ​"Once the pain impulse reaches the spinal​ cord, the neurotransmitters speed up the signal for sharp​ pain." D. ​"Both the alpha and beta fibers in the spinal cord work together to signal sharp​ pain."

B. ​"Alpha fibers are wrapped in myelin that speed up the signal resulting in the sensation of sharp​ pain."

A client with anxiety and insomnia asks the nurse about the benefits of treatment with a benzodiazepine. Which response should the nurse provide the​ client? A. ​"Benzodiazepines are the most​ effective." B. ​"Benzodiazepines have the lowest risk of dependency and​ tolerance." C. ​"Benzodiazepines are most​ affordable." D. ​"Benzodiazepines are most likely to be covered under insurance​ premiums."

B. ​"Benzodiazepines have the lowest risk of dependency and​ tolerance."

A client asks the nurse about bipolar disorder. Which response should the nurse​ provide? A. ​"Bipolar disorder is a type of depression that includes attention deficit disorder​ symptoms." B. ​"Bipolar disorder means you have cycles of depression as well as​ hyperactivity, or​ mania." C. ​"Bipolar disorder is just another type of​ depression, except the depression occurs in​ cycles." D. ​"Bipolar disorder just means that the mood alternates with the​ seasons, and becomes worse in the​ winter."

B. ​"Bipolar disorder means you have cycles of depression as well as​ hyperactivity, or​ mania."

A client has returned to the clinic for​ follow-up after being prescribed an adrenergic blocking medication for benign prostatic hypertrophy​ (BPH.) Which assessment questions should the nurse​ ask? Select all that apply. A. ​"What color is your​ urine?" B. ​"Do you have any difficulty starting to​ urinate?" C. ​"Do you have the feeling that your bladder is​ full?" D. ​"When was your last bowel​ movement?" E. ​"Have you noticed being more hungry than​ usual?"

B. ​"Do you have any difficulty starting to​ urinate?" C. ​"Do you have the feeling that your bladder is​ full?"

The nurse has provided education for a client diagnosed with epilepsy. Which statement made by the client indicates an understanding of the​ information? Select all that apply. A. ​"Excessive stress levels cause disruptions in how the brain receives​ oxygen, leading to​ epilepsy." B. ​"Epilepsy may be caused by a head​ injury." C. ​"Eating disorders, like anorexia​ nervosa, increase the risk for developing​ epilepsy." D. ​"A stroke, or brain​ attack, could increase the risk for developing​ epilepsy." E. ​"With some cases of​ epilepsy, the cause is never​ determined."

B. ​"Epilepsy may be caused by a head​ injury." D. ​"A stroke, or brain​ attack, could increase the risk for developing​ epilepsy." E. ​"With some cases of​ epilepsy, the cause is never​ determined."

A client is to be prescribed an antidepressant. Which question is most important when performing the initial​ assessment? A. ​"How much alcohol do you consume during the​ week?" B. ​"Have you had any thoughts about killing​ yourself?" C. ​"Are you allergic to any​ medications?" D. ​"How long have you been​ depressed?"

B. ​"Have you had any thoughts about killing​ yourself?"

A client has been receiving clozapine​ (Clozaril) for the treatment of schizophrenia. Which statement made by the client indicates the prescription is​ effective? A. ​"I will start going to group​ therapy." B. ​"I am not hearing the voices​ anymore." C. ​"I promise not to skip breakfast​ anymore." D. ​"I think I am ready for​ discharge, as I feel​ better."

B. ​"I am not hearing the voices​ anymore."

The client receives methyldopa​ (Aldomet). Which statement below indicates the patient is experiencing side​ effects? A. ​"Will you check my​ pupils? I​ can't see very well at​ all." B. ​"I am so​ anxious; I really need to walk around the​ room." C. ​"I am so​ thirsty; will you please bring me another pitcher of​ water?" D. ​"I feel so sleepy that I​ don't think I can eat my​ dinner."

B. ​"I am so​ anxious; I really need to walk around the​ room."

The nurse has completed the education for a client prescribed buspirone​ (BuSpar). Which statement made by the client indicates further teaching is​ required? A. ​"This prescription should be taken on a regular basis for it to help​ me." B. ​"I can take this prescription when I feel anxious and it will relax​ me." C. ​"I don't need to worry about becoming dependent on this​ prescription." D. ​"The side effects I might experience include​ dizziness, headache, and​ drowsiness."

B. ​"I can take this prescription when I feel anxious and it will relax​ me."

Which statements made by a client scheduled for an operative procedure for which isoflurane​ (Forane) will be administered should the nurse immediately report to the healthcare​ provider? Select all that apply. A. ​"I stopped taking my vitamin C supplement last week because it upset my​ stomach." B. ​"I don't think my St.​ John's Wort is helping my​ depression." C. ​"I have not eaten anything since before my CT scan​ yesterday." D. ​"My brother ran a high fever and had seizures after his​ surgery." E. ​"My last dose of levodopa was​ yesterday."

B. ​"I don't think my St.​ John's Wort is helping my​ depression." D. ​"My brother ran a high fever and had seizures after his​ surgery."

The nurse has provided the education for the client prescribed atenolol​ (Tenormin). Which statement made by the client indicates an understanding of the​ information? A. ​ "I cannot take this drug if I develop​ glaucoma." B. ​"I need to take my pulse every​ day." C. ​"I cannot continue to have my morning cup of​ coffee." D. ​"If I have any side​ effects, I will stop the​ medication."

B. ​"I need to take my pulse every​ day."

the nurse has completed medication education for a client prescribed phenelzine​ (Nardil). Which statements made by the client indicate an understanding of the​ teaching? Select all that apply. A. ​"I am really going to miss my morning coffee and sweet​ roll." B. ​"I'll have to give up my beer at the football​ games." C. ​"I can't eat fried chicken and​ gravy." D. ​"I am not supposed to have processed meats or​ cheese." E. ​"I really​ shouldn't eat at a​ restaurant; too many foods are on my restricted​ list."

B. ​"I'll have to give up my beer at the football​ games." D. ​"I am not supposed to have processed meats or​ cheese."

A client asks the nurse how morphine will control pain. Which response should the nurse provide the​ client? A. ​"Inhibits the primary pain neurotransmitters in your​ brain." B. ​"Interacts with receptors in your body that produce​ analgesia." C. ​"Promotes the primary pleasure neurotransmitters in your​ brain." D. ​"Stimulates the receptors that secrete endorphins in your​ brain."

B. ​"Interacts with receptors in your body that produce​ analgesia."

The nurse has completed medication education about pyridostigmine​ (Mestinon), an indirect cholinergic​ drug, for a client with myasthenia gravis. The nurse determines that learning has occurred when the client makes which​ statement? A. ​"I will need to increase my fluid intake with this​ medication." B. ​"It is really important to take my medication on​ time." C. ​"My heart may beat slower while I am on this​ drug." D. ​"I must take this medication immediately before eating a full​ meal."

B. ​"It is really important to take my medication on​ time."

Which statement made by a client prescribed acetylsalicylic acid​ (aspirin) indicates the client is experiencing an adverse​ reaction? A. ​"I have to get up a lot at night to​ urinate." B. ​"My stools have been dark in​ color." C. ​"Bright lights give me a​ headache." D. ​"My nose is stuffed​ up."

B. ​"My stools have been dark in​ color."

A client taking phenobarbital​ (Luminal) for seizure control asks the nurse how the prescription can control seizures. Which response should the nurse provide the​ client? A. ​"Phenobarbital (Luminal) stops seizures by decreasing the sodium in​ brain, which is responsible for the​ seizures." B. ​"Phenobarbital (Luminal) stops seizures by increasing a chemical called GABA that calms down the excitability in the​ brain." C. ​"Phenobarbital (Luminal) stops seizures by increasing a chemical called glutamate that calms down the excitability in the​ brain." D. ​"Phenobarbital (Luminal) stops seizures by decreasing the calcium in the​ brain, which is responsible for the​ seizures."

B. ​"Phenobarbital (Luminal) stops seizures by increasing a chemical called GABA that calms down the excitability in the​ brain."

A client with epilepsy prescribed Phenobarbital​ (Luminal) tells the nurse she is planning a pregnancy. Which statement should the nurse provide the​ client? A. ​"Please talk to your​ doctor; you will need a safer drug like valproic acid​ (Depakene)." B. ​"Please talk to your​ doctor; this drug is contraindicated in​ pregnancy." C. ​"Your medication dose will need to be increased during your​ pregnancy." D. ​"Your medication dose will need to be decreased during your​ pregnancy."

B. ​"Please talk to your​ doctor; this drug is contraindicated in​ pregnancy."

A client has been prescribed prazosin​ (Minipress). Which information should the nurse​ provide? Select all that apply. A. ​"Stay out of the sun until you determine if you become​ sun-sensitive." B. ​"Take this medication just before you go to​ bed." C. ​"This medication may make you​ dizzy." D. ​"This medication may slow your heart rate​ noticeably." E. ​"Do not take this medication with​ milk."

B. ​"Take this medication just before you go to​ bed." C. ​"This medication may make you​ dizzy."

A client asks the nurse why he or she will receive IV anesthesia before inhaled anesthesia. Which response should the nurse​ provide? A. ​"The IV is used to administer large amounts of solutions required by the​ anesthesia." B. ​"The IV anesthesia will produce a rapid​ unconsciousness." C. ​"The IV is used to administer a volatile liquid that will help maintain a deep​ sleep." D. ​"The IV anesthesia is easier to reverse if adverse effects​ occur."

B. ​"The IV anesthesia will produce a rapid​ unconsciousness."

The educator is discussing pain transmission with the nurse. Which statement made by the nurse indicates an understanding of the​ process? A. ​"The transmission process begins with the sensory neurons in the spinal​ cord." B. ​"The transmission process begins with the​ nociceptors." C. ​"The transmission process begins with the A and C​ fibers." D. ​"The transmission process begins with the activation of substance​ P."

B. ​"The transmission process begins with the​ nociceptors."

Which non pharmacological interventions should the nurse recommend for a client that is having difficulty​ sleeping? Select all that apply. A. ​"Make certain your bedroom is nice and​ warm." B. ​"There are alternative methods to treat​ insomnia, such as​ yoga, meditation, and massage​ therapy." C. ​"Eating a large meal at bedtime will help induce​ sleep." D. ​"Avoid caffeinated​ beverages, nicotine, and alcohol immediately prior to​ bedtime." E. ​"Play your favorite type of music when you go to​ sleep."

B. ​"There are alternative methods to treat​ insomnia, such as​ yoga, meditation, and massage​ therapy." D. ​"Avoid caffeinated​ beverages, nicotine, and alcohol immediately prior to​ bedtime."

A client asks the nurse about the characteristics of absence seizures. Which explanation should the nurse provide the​ client? A. ​"Absence seizures are characterized by twitching of the arms and​ legs." B. ​"This type of seizure manifests is characterized by staring into space for a few​ seconds." C. ​"This type of seizure is similar to drop​ attacks." D. ​"Absence seizures are basically the same kind of seizures as grand​ mal, but they are less​ frequent."

B. ​"This type of seizure manifests is characterized by staring into space for a few​ seconds."

A client prescribed lithium​ (Eskalith) asks the nurse why they should continue to use salt. Which response should the nurse provide the​ client? A. ​"You should use sea salt to avoid lithium​ (Eskalith) toxicity." B. ​"You must use table salt or your kidneys will retain lithium​ (Eskalith), and you will become​ toxic." C. ​"The amount of salt is not​ important; just increase your table salt if you notice signs of lithium​ (Eskalith) toxicity." D. ​"Salt is very important to avoid lithium​ (Eskalith) toxicity, but not as important as drinking 1 to 1.5 L of water per​ day."

B. ​"You must use table salt or your kidneys will retain lithium​ (Eskalith), and you will become​ toxic."

A client diagnosed with epilepsy ask the nurse why he or she will have to take a prescription. Which response should the nurse provide the​ client? A. ​"You might not need​ prescription; the epilepsy may be controlled with a ketogenic​ diet." B. ​"You will need to take a prescription on a continual basis to control the​ seizures." C. ​"You will need a prescription for a little while to cure the​ seizures." D. ​"The prescription will decrease the intensity of the​ seizures."

B. ​"You will need to take a prescription on a continual basis to control the​ seizures."

Which food items should the nurse advise a client taking a monoamine oxidase inhibitor​ (MAOI) to​ avoid? A. ​Spinach, rice, and venison B. ​Chocolate, wine, and fava beans C. Orange​ juice, cottage​ cheese, and turkey D. Spring​ water, ice​ cream, and salmon

B. ​Chocolate, wine, and fava beans

A nurse is providing teaching to a client who has a new prescription for fluoxetine. Which of the following instructions should the nurse include? A. "Weight yourself twice a month." B. "If you have persistent headaches, let the provider know." C. "Reduce your daily sodium intake." D. "If your symptoms don't improve in 10 days, you will need a higher dosage."

B. "If you have persistent headaches, let the provider know."

A nurse is caring for a client who has schizoaffective disorder and who has been prescribed chlorpromazine IV. Which of the following client findings should the nurse monitor after administering the drug? A. BUN level B. Blood pressure C. Urine specific gravity D. pedal pulse strength

B. Blood pressure

The nurse has provided parental education of the treatment plan for the spasticity their​ 2-year-old is experiencing. Which statements by the parents indicate additional teaching is​ required? Select all that apply... A. ​"At some​ point, our child may require surgery to correct this​ spasticity." B. ​"As long as we continue our​ child's medications, the spasticity can be​ controlled." C. ​"Our physical therapy sessions should focus on flexing our​ child's muscles." D. ​"We should repeat the exercises several times with each muscle​ group." E. ​"It is best to give our child a rest from physical therapy by skipping 1 week a​ month."

B. C. and E.

The nurse is preparing to discharge an older adult prescribed cyclobenzaprine​ (Flexeril) 10 mg three times per day. The prescription is written for 90 tablets and with three refills available. Which information should the nurse recognize requires collaboration with the healthcare​ provider? Select all that apply... A. The dosage amount is too low for the type of injury this patient sustained. B. Cyclobenzaprine should be used with caution in those over 65. C. If taken as​ directed, the patient would be able to take the medication for 120 days. D. Cyclobenzaprine is not effective for back pain. E. Cyclobenzaprine should not be taken with food.

B. C. and E.

A nurse is providing teaching for a client who has a prescription for phenelzine. The nurse should instruct the client to avoid tyramine-enriched foods because of an increased risk for which of the following adverse reactions? A. Respiratory depression B. Hypertensive crisis C. Neuroleptic malignant syndrome D. Serotonin syndrome

B. Hypertensive crisis

A nurse is providing teaching to a client who has a prescription for chlorpromazine. Which of the following instructions should the nurse include? A. Stop taking the drug immediately with the first sign of a sore throat B. Wear sunscreen when exposed to sunlight C. Take the drug with food to reduce gastrointestinal distress D. Take the drug in the morning to prevent nocturia

B. Wear sunscreen when exposed to sunlight

A client with alcoholism prescribed disulfiram​ (Antabuse) tells the nurse that he or she stopped taking the prescription a week prior and have begun drinking again. Which assessment finding should the nurse​ anticipate? A. Confusion and fine tremors B. An absence of symptoms C. ​Headache, nausea, and vomiting D. Severe hyperglycemia

C

A parent expressed concern that his or her adolescent son is up late at​ night, is​ listless, and frequently uses eye drops for his bloodshot eyes. Which statement should the nurse include in the discussion with the​ parent? A. ​"Sounds like he is using​ cocaine." B. ​"He might be​ depressed; insomnia is common with​ depression." C. ​"He could be smoking​ marijuana." D. ​"Your son could be studying too​ hard."

C

The nurse is providing education to a client that has smoked 1 pack of cigarettes daily for the past few years and is currently prescribed oral contraceptives. Which statement should the nurse include in the​ teaching? A. ​"The nicotine will decrease the effectiveness of your birth control​ pills." B. ​"You are at a higher risk for emphysema than general​ smokers." C. ​"You are at a higher risk for a heart attack than​ nonsmokers." D. ​"You are at a higher risk for developing diabetes than smokers who do not use birth control​ pills."

C

Which symptoms should the nurse anticipate are associated with alcohol​ withdrawal? A. ​Anxiety, seizures, hearing​ loss, and alopecia B. Abdominal​ pain, chills, pupil​ dilation, and lethargy C. ​Tremors, anxiety,​ confusion, and delirium D. ​Lethargy, constipation, light​ sensitivity, and weight loss

C

A client tells the nurse he or she is experiencing muscle discomfort resulting from weight lifting. Which prescription should the nurse anticipate will be included in the​ treatment? A. Clonidine​ (Catapres) B. Baclofen​ (Lioresal) C. Ibuprofen D. Clonazepam​ (Klonopin)

C.

A client tells the nurse they are awakened during the night with leg and foot cramps. Which response should the nurse provide the​ client? A. ​"Take a warm bath before going to​ bed." B. ​"Apply heat to relieve the​ cramping." C. ​"Increase your intake of vitamin B6​." D. ​"You may need a muscle​ relaxant."

C.

The nurse is creating a plan of care for a client prescribed cyclobenzaprine experiencing muscle spasms. Which outcome should the nurse​ anticipate? A. Improved​ self-concept. B. Stabilized vital signs. C. Uninterrupted sleep. D. Increased bladder tone.

C.

Which characteristics of sedation describe a client that has been administered general​ anesthesia? A. A sleeping state that can be awakened easily B. An unconscious​ state, without analgesia C. A total loss of body movements D. A conscious but sleepy state of being

C. A total loss of body movements

A client prescribed haloperidol​ (Haldol) is observed making wormlike movements with their tongue. Which should the nurse associate this finding​ with? A. A symptom of the underlying psychosis B. An indication that the patient has not taken the prescription C. An adverse effect likely caused by the medication D. A normal reaction to the medication that will resolve within a few weeks

C. An adverse effect likely caused by the medication

A client with type I diabetes mellitus has been prescribed phenytoin​ (Dilantin). Based on the​ client's medical​ history, which should the nurse anticipate to be included in the treatment​ plan? A. A decrease in the​ patient's insulin. B. Implement fall precautions. C. An increase the​ patient's insulin. D. Monitor coagulation studies.

C. An increase the​ patient's insulin.

The nurse is preparing to assist a healthcare provider suture a laceration on a​ 2-year-old child. Prior to the​ procedure, which initial intervention should the nurse​ anticipate? A. Requesting the parents to leave the room prior to the procedure. B. Wrapping the child in a blanket for restraint and safety during suturing. C. Applying a topical anesthetic prior to administering a local anesthetic with a needle. D. Administering a small dose of lorazepam​ (Ativan).

C. Applying a topical anesthetic prior to administering a local anesthetic with a needle.

A client with gastroesophageal reflux disease receives lidocaine viscous prior to an endoscopy. Which is the​ nurse's priority assessment after the​ procedure? A. Assess for a headache. B. Assess for nausea and vomiting. C. Assess for a return of the gag reflex. D. Assess for any damage to the teeth or gums.

C. Assess for a return of the gag reflex.

The nurse is caring for a client suspected of overdosing on donepezil​ (Aricept). Which prescription should the nurse prepare to​ administer? A. Physostigmine B. Gastric lavage with charcoal C. Atropine D. Epinephrine

C. Atropine

For which client is acetaminophen​ (Tylenol) contraindicated? A. A​ 2-year-old with a high fever B. A​ 65-year-old with osteoarthritis C. A​ 55-year old who drinks alcohol D. A​ 19-year-old with a bladder infection

C. A​ 55-year old who drinks alcohol

Which is an adrenergic receptor associated with the sympathetic nervous​ system? A. Norepinephrine B. Dopamine C. Beta D. Muscarinic

C. Beta

A client has been prescribed isoproterenol​ (Isuprel). The nurse understands that which receptors will be​ stimulated? Select all that apply. A. Alpha1 B. Alpha2 C. Beta1 D. Beta2 E. Beta3

C. Beta1 D. Beta2

The nurse has completed the dietary education for a patient prescribed an antipsychotic. Which food identified by the client has to be eliminated indicates an understanding of the​ teaching? A. Meats with a​ high-fat content B. Refined breads and desserts C. Beverages with caffeine D. Shellfish and peanuts

C. Beverages with caffeine

The nurse asks the educator how local anesthetics work. Which response should the educator provide the​ nurse? A. Occupy potassium receptors. B. Increase nerve impulse transmission. C. Block the sodium channels. D. Enhancing the influx of calcium into the cell.

C. Block the sodium channels.

A student nurse asks a nurse how atropine​ (Atropair) increases heart rate. What is the​ nurse's best​ response? A. Blocking the beta receptors of the parasympathetic nervous system. B. Directly stimulating the sympathetic nervous system. C. Blocking the effects of acetylcholine by occupying muscarinic receptors. D. Potentiating the effects of acetylcholine on nicotinic receptors.

C. Blocking the effects of acetylcholine by occupying muscarinic receptors.

The nurse is caring for an older adult prescribed benztropine​ (Cogentin). For which adverse effect should the nurse monitor the​ patient? A. Confusion B. Diarrhea C. Bradycardia D. Agitation

C. Bradycardia

The client receiving valproic acid​ (Depakene) for treatment of seizures has been taking 81 mg aspirin daily. For which should the nurse monitor the​ client? A. Increased seizure activity. B. ​Stevens-Johnson syndrome. C. Bruising of the skin. D. Migraine headaches.

C. Bruising of the skin.

Which should the nurse recognize are the traditionally prescribed drug classifications used to treat​attention-deficit/hyperactivity disorder​ (ADHD)? A. Sympathomimetics B. Parasympathomimetics C. CNS stimulants D. CNS depressants

C. CNS stimulants

The nurse is reviewing the prescription history for a client receiving levodopa​ (Stalevo). Which routine prescription is most​ concerning? A. Aspirin B. Metformin C. Calcium antacid D. Synthroid

C. Calcium antacid

A client is brought to the emergency department suspected of overdosing on imipramine​ (Tofranil). Which is the​ nurse's priority​ assessment? A. Neurological function B. Liver function C. Cardiac function D. Renal function

C. Cardiac function

The nurse is providing education for a client about the characteristics of​ Alzheimer's disease. Which information should the nurse​ include? Select all that apply. A. More women than men have​ Alzheimer's disease. B. The disease is characterized by the degeneration of the neurons. C. Chronic inflammation of the brain may be a cause of the disease. D. The disease mainly affects people over 50 years of age.

C. Chronic inflammation of the brain may be a cause of the disease.

A student nurse is reviewing the autonomic nervous system. The student understands that which substance would inhibit the function of the autonomic nervous​ system? A. Drugs that bind and then stimulate the postsynaptic neuron. B. Drugs that prohibit neurotransmitter reuptake. C. Drugs that prevent the storage of neurotransmitter in vesicles. D. Drugs that increase neurotransmitter synthesis.

C. Drugs that prevent the storage of neurotransmitter in vesicles.

Which type of local anesthesia should the nurse anticipate will be administered to a laboring​ client? A. Infiltration B. Topical C. Epidural D. Nerve block

C. Epidural

Which should the nurse understand is the purpose of the epinephrine added to local​ anesthetics? A. Promotion of patient relaxation. B. Promotion of blood flow to the area. C. Extension of the duration of the action of the local anesthetic. D. Decrease the amount of pain experienced.

C. Extension of the duration of the action of the local anesthetic.

A client tells the nurse he or she worries about everything all​ day, feels​ confused, restless. Which should the nurse suspect the client is​ experiencing? A. Panic disorder B. ​Post-traumatic stress disorder​ (PTSD) C. Generalized anxiety D. Social anxiety

C. Generalized anxiety

Which prescription should the nurse anticipate to be included in the treatment of a client experiencing mild migraine​ headaches? A. Amitriptyline​ (Elavil) B. Ergotamine​ (Ergostat) C. Ibuprofen D. Sumatriptan​ (Imitrex)

C. Ibuprofen

Which should the nurse understand is the action of the pharmacotherapy used to treat​ Parkinson's disease? A. Increase the amount of dopamine and acetylcholine. B. Reduce the amount of dopamine and increase the amount of acetylcholine. C. Increase the amount of dopamine and reduce the amount of acetylcholine. D. Reduce the amount of dopamine and acetylcholine.

C. Increase the amount of dopamine and reduce the amount of acetylcholine.

A client who takes benztropine​ (Cogentin) has developed constipation. Which interventions should the nurse​ implement? Select all that apply. A. Instruct the client to administer an enema. B. Encourage the client to take docusate​ (Colace) daily. C. Instruct the client to increase their fluid intake. D. Encourage the client to walk daily E. Instruct the client to discontinue their prescription until their bowel function returns to normal.

C. Instruct the client to increase their fluid intake. D. Encourage the client to walk daily

Which should the nurse recognize is a benefit of obtaining a thorough history from a client who is experiencing​ anxiety? A. The type of counseling can be prescribed based on the history. B. The category of anxiety disorder can be identified. C. Pharmacotherapy can be prescribed. D. The treatment plan can be created.

C. Pharmacotherapy can be prescribed.

A client asks the nurse which prescriptions are commonly used to treat insomnia. Which pharmacological agents should the nurse discuss with the​ client? Select all that apply. A. Diphenhydramine​ (Benadryl) B. Valerian root C. Ramelteon​ (Rozerem) D. Flurazepam​ (Dalmane) E. Zolpidem​ (Ambien)

C. Ramelteon​ (Rozerem) D. Flurazepam​ (Dalmane) E. Zolpidem​ (Ambien)

Which classification of prescription is often used as a​ first-line treatment for depression due to their safety​ profile? A. Monoamine oxidase inhibitors​ (MAOIs) B. ​Beta-adrenergic blockers C. Selective serotonin reuptake inhibitors​ (SSRIs) D. Tricyclic antidepressants​ (TCAs)

C. Selective serotonin reuptake inhibitors​ (SSRIs)

A client has taken different prescriptions over the past two years that have been unsuccessful in treating his or her insomnia. Which condition should the nurse suspect the client may be​ experiencing? A. Personality disorder B. Addiction C. Sleep apnea D. Anxiety disorder

C. Sleep apnea

The nurse is preparing to provide education for a client prescribed carbidopa​ (Sinemet). Which information should the nurse​ include? A. Take the medication with a protein food. B. Avoid drinking caffeinated beverages. C. Take the medication on an empty stomach. D. Take the medication with meals.

C. Take the medication on an empty stomach.

Which statement is accurate concerning drugs classified as adrenergic​ antagonists? A. They are also known as anticholinergics. B. They will stimulate the sympathetic nervous system. C. The actions are the opposite of those of sympathomimetics. D. The actions will block the neurotransmitter acetylcholine.

C. The actions are the opposite of those of sympathomimetics.

Which is the priority outcome for a​ 6-year-old child that has been prescribed methylphenidate​ (Ritalin)? A. The child will be able to complete​ age-appropriate chores at home. B. The child will avoid altercations with peers. C. The child will use​ age-appropriate play with peers. D. The child will maintain weight within norms for this age group.

C. The child will use​ age-appropriate play with peers.

A client with schizophrenia sits quietly in a chair and minimally responds to the environment. Which of the following assessment findings should the nurse​ interpret? A. The client is most likely very depressed. B. The client is experiencing positive symptoms. C. The client is experiencing negative symptoms. D. The client is most likely hearing voices.

C. The client is experiencing negative symptoms.

The nurse has completed the education for a client prescribed lithium​ (Eskalith). Which should the nurse anticipate is a priority outcome of the​ treatment? A. The client will engage in activities of daily living and report enjoyment with them. B. The client will be able to work a normal work schedule and will receive adequate sleep. C. The client will identify signs of lithium​ (Eskalith) toxicity and verbalize measures to avoid it. D. The client will report stabilization of​ mood, including absence of mania or depression.

C. The client will identify signs of lithium​ (Eskalith) toxicity and verbalize measures to avoid it.

The nurse has provided teaching for a client prescribed clonazepam​ (Klonopin). Which is an important outcome when evaluating the​ client's understanding of the​ teaching? A. The client will verbalize the importance of dietary restrictions related to this drug. B. The client will verbalize the signs of developing​ Stevens-Johnson rash. C. The client will verbalize the consequences of abruptly stopping the drug. D. The client will verbalize the importance of having routine laboratory work.

C. The client will verbalize the consequences of abruptly stopping the drug.

The nurse is preparing to assess a client suspected of experiencing tardive dyskinesia. Which findings should the nurse anticipate that support the​ diagnosis? A. Severe muscle spasms of the​ back, neck, and tongue B. ​Tremor, muscle​ rigidity, and a shuffling gait C. Unusual facial movements and lip smacking D. An inability to rest or​ relax, and restlessness

C. Unusual facial movements and lip smacking

A client tells the nurse he or she has a​ dull, aching pain in his or her lower back. Which type of pain should the nurse recognize the client is​ experiencing? A. Neuropathic B. Somatic C. Visceral D. Acute

C. Visceral

The nurse is preparing the education for a client prescribed a selective serotonin reuptake inhibitors​ (SSRI). Which adverse effects should the nurse include in the​ teaching? A. Drowsiness and coma B. Dry mouth and urine retention C. Weight gain and sexual dysfunction D. Headache and nausea

C. Weight gain and sexual dysfunction

A client with a generalized anxiety disorder asks the nurse if he or she will need medication for treatment. Which response should the nurse provide the​ client? A. ​"A prescription is a common treatment for clients with anxiety​ disorders." B. ​"A prescription is necessary​ initially; followed by​ therapy." C. ​"A prescription is necessary when anxiety interferes with your quality of​ life." D. ​"You most likely will not require a​ prescription."

C. ​"A prescription is necessary when anxiety interferes with your quality of​ life."

A parent of an​ 8-year-old child prescribed methylphenidate​ (Ritalin) tells the nurse the child will not eat. Which response should the nurse provide the​ parent? A. ​"It sounds like your child may be experiencing​ depression." B. ​"Your child may require an appetite​ stimulant." C. ​"Administer the prescription after meals and encourage snacks between​ meals." D. ​"I will get you a consult with a​ nutritionist."

C. ​"Administer the prescription after meals and encourage snacks between​ meals."

A client asks the nurse if there are any​ over-the-counter herbs or nonprescription medications that can be used to improve insomnia. Which response should the nurse provide the​ client? ​Select all that apply. A. ​"Ginger root is commonly taken to improve​ sleep." B. ​"Ginkgo is an herb commonly taken to improve​ sleep." C. ​"Diphenhydramine (Benadryl) and doxylamine are​ over-the-counter meds sometimes taken to produce​ drowsiness." D. ​"Valerian and melatonin are herbs commonly taken to improve​ sleep." E. ​"Kava is an herb taken to improve​ sleep."

C. ​"Diphenhydramine (Benadryl) and doxylamine are​ over-the-counter meds sometimes taken to produce​ drowsiness." D. ​"Valerian and melatonin are herbs commonly taken to improve​ sleep."

A client has been prescribed several different prescriptions for seizure control without any improvement. Which initial question should the nurse ask the​ client? A. ​"Have you noticed any decrease in the amount of seizures you​ have?" B. ​"Have you thought about taking a nontypical​ prescription?" C. ​"Do you take your medication as​ prescribed?" D. ​"Do you drink​ alcohol?"

C. ​"Do you take your medication as​ prescribed?"

The nurse provided education for a client prescribed risperidone​ (Risperdal). Which statement made by the client indicates an understanding of the​ teaching? A. ​"I must call my healthcare provider if I start to have a lot of​ nightmares." B. ​"I must call my healthcare provider if I start to lose a lot of​ weight." C. ​"I must have my blood work done while taking this​ medication." D. ​"I must call my healthcare provider if I notice any a metallic taste in my​ mouth."

C. ​"I must have my blood work done while taking this​ medication."

The nurse has provided the education to the parent of a young child prescribed valproic acid​ (Depakene) syrup. Which statement made by the parent indicates an understanding of the​ teaching? A. ​"I can give this on a full stomach or an empty​ stomach." B. ​"If my child gets a headache from​ this, I can administer a baby​ aspirin." C. ​"I should not mix this with carbonated​ beverages." D. ​"I can still give the clonazepam​ (Klonopin) prescribed by the psychiatrist for​ sleep."

C. ​"I should not mix this with carbonated​ beverages."

The nurse has provided education for a client prescribed ethosuximide​ (Zarontin). For which information offered by the client should the nurse notify the healthcare​ provider? Select all that apply. A. ​"Did I mention that I used to take phenobarbital for my​ seizures?" B. ​"I forgot to tell the doctor that I am allergic to​ penicillin." C. ​"I take ginkgo to help improve my​ memory." D. ​"My husband and I plan to have a baby in a couple of​ years." E. ​"My husband and I are leading a​ 20-mile bicycle tour next weekend for the company we have just​ started."

C. ​"I take ginkgo to help improve my​ memory." E. ​"My husband and I are leading a​ 20-mile bicycle tour next weekend for the company we have just​ started."

The nurse has provided teaching about the use of an auto injection of epinephrine. Which statement made by the client indicates further teaching is​ required? A. ​"I will make sure I have my​ auto-injector available at all​ times." B. ​"I will use my​ auto-injector immediately if I think I am having an allergic​ reaction." C. ​"I will call 911 after I inject the​ epinephrine." D. ​"I will notify my healthcare provider after I inject the​ epinephrine."

C. ​"I will call 911 after I inject the​ epinephrine."

A client prescribed sertraline​ (Zoloft) calls the clinic reporting increased moodiness and feeling disconnected from life. Which response should the nurse provide the​ client? A. ​"Try taking diphenhydramine​ (Benadryl) to help you​ relax." B. ​"Do not take your prescription until your mood​ improves." C. ​"It is important that you come into the clinic​ today." D. ​"Double the prescribed dose of sertraline​ (Zoloft) today."

C. ​"It is important that you come into the clinic​ today."

A client asks the nurse how methadone​ (Dolophine) can help with a heroin dependence. Which response should the nurse​ provide? A. ​"Methadone (Dolophine) will cause illness heroin is​ used." B. ​"Methadone (Dolophine) can cure an addiction to​ heroin." C. ​"Methadone (Dolophine) does not cause euphoria like heroin​ does." D. ​"Methadone (Dolophine) causes an allergic reaction to​ heroin."

C. ​"Methadone (Dolophine) does not cause euphoria like heroin​ does."

The student nurse asks the nursing​ instructor, "Do medications actually make more​ neurotransmitters?" What is the best response by the nursing​ instructor? A. ​"Yes, but the newer drugs that do this have some serious side​ effects." B. ​"Yes, some of the newer medications are very good at doing​ this." C. ​"No, medications can only increase or decrease the action of​ neurotransmitters." D. ​"No, but medications can heal diseases of the autonomic nervous​ system."

C. ​"No, medications can only increase or decrease the action of​ neurotransmitters."

A client with epilepsy who has been seizure free and prescribed phenytoin​ (Dilantin) asks the nurse why he or she needs to still have blood testing. Which response should the nurse provide the​ client? A. ​"Phenytoin (Dilantin) can cause​ blood-thinning in some​ patients." B. ​"Phenytoin (Dilantin) can deplete your system of​ potassium." C. ​"Phenytoin (Dilantin) has a very narrow range between a therapeutic dose and a toxic​ dose." D. ​"Phenytoin (Dilantin) can cause​ Stevens-Johnson syndrome, which can be detected through blood​ testing."

C. ​"Phenytoin (Dilantin) has a very narrow range between a therapeutic dose and a toxic​ dose."

A client prescribed quetiapine​ (Seroquel) asks the nurse how the prescription works. Which response should the nurse provide the​ client? A. ​"Quetiapine (Seroquel) increases norepinephrine in the​ brain, which decreases the feelings of​ depression." B. ​"Quetiapine (Seroquel) increases dopamine in the​ brain, which helps to think more​ clearly." C. ​"Quetiapine (Seroquel) decreases dopamine in the​ brain, which decreases your​ symptoms." D. ​"Quetiapine (Seroquel) decreases norepinephrine in the​ brain, which decreases your auditory​ hallucinations."

C. ​"Quetiapine (Seroquel) decreases dopamine in the​ brain, which decreases your​ symptoms."

A client taking amitriptyline​ (Elavil) for 2 weeks tells the nurse the prescription is not working. Which response should the nurse provide the​ client? A. ​"You might still feel​ depressed, but you are looking much​ better." B. ​"You may need additional​ pharmacotherapy." C. ​"The prescription can take several weeks to have an​ effect." D. ​"This may not be the best prescription for​ you; I will notify your healthcare​ provider."

C. ​"The prescription can take several weeks to have an​ effect."

A client with multiple sclerosis asks the nurse how Dalfampridine​ (Ampyra) will help them. Which response should the nurse​ provide? A. ​"Dalfampridine will decrease the number of mood​ alterations." B. ​"The prescription will disrupt the visual disturbances that occur with​ MS." C. ​"The prescription will improve your ability to​ walk." D. ​"Dalfampridine will lessen the cognitive deficits that occur with​ MS."

C. ​"The prescription will improve your ability to​ walk."

A client experiencing nervousness asks the nurse why escitalopram was prescribed. Which response should the nurse provide the​ client? A. ​"You are really​ depressed; it is just manifested as anxiety. These medications are safer than​ benzodiazepines." B. ​"The two disorders are​ similar, and if you treat​ depression, the anxiety goes​ away." C. ​"The same brain chemicals are involved with anxiety and​ depression." D. ​"Your doctor thinks that this is the best treatment for your​ anxiety, and these medications are safer than​ benzodiazepines."

C. ​"The same brain chemicals are involved with anxiety and​ depression."

A parent tells the nurse they are concerned because their child is easily distracted in​ school, cannot complete assignments on​ time, and interrupts other children while they are speaking. Which response should the nurse provide the​ parent? A. ​"This sounds like your child is​ depressed; depression looks different in children and is very​ serious." B. ​"This sounds like bipolar​ disorder; you might want to have your child tested by a child​ psychiatrist." C. ​"This could be​ attention-deficit/hyperactivity disorder​ (ADHD); you might want to have your child​ tested." D. ​"This sounds like typical​ 7-year-old behaviors to​ me; if they do not​ resolve, have your child​ tested."

C. ​"This could be​ attention-deficit/hyperactivity disorder​ (ADHD); you might want to have your child​ tested."

A client who has used an adrenergic nasal spray for 2 weeks states to the​ nurse, "I am more stuffed up now than I was when I was​ sick." Which information should the nurse provide the​ patient? Select all that apply. A. ​"You are having an allergic reaction to the nasal spray. Stop using it​ immediately." B. ​"You are having an allergic reaction to the nasal spray. Stop using it​ immediately." C. ​"Try increasing the amount of fluids you are​ drinking." D. ​"Switch to a​ saline-based nasal​ spray." E. ​"Continue to use your current nasal spray until the congestion goes​ away."

C. ​"Try increasing the amount of fluids you are​ drinking." D. ​"Switch to a​ saline-based nasal​ spray."

The nurse is obtaining a history from a client that experienced an atonic seizure. Which information is the most important for the nurse to​ obtain? A. ​"Do you have a history of​ infertility?" B. ​"Have you ever been​ pregnant?" C. ​"What method of birth control do you​ use?" D. ​"Are your periods​ regular?"

C. ​"What method of birth control do you​ use?"

Which question should the nurse ask a postsurgical client prior to the administration of an opioid​ analgesic? A. ​"Have you ever been addicted to prescription pain​ prescriptions?" B. ​"Would you like me to help you change your position for​ comfort?" C. ​"Would you please rate your pain on a scale of 1 to​ 10?" D. ​"Do you understand how this pain prescription​ works?"

C. ​"Would you please rate your pain on a scale of 1 to​ 10?"

A pregnant client requiring a skin biopsy expresses concern about the anesthesia that will be administered. Which response should the nurse​ provide? A. ​"There are newer general anesthetics available that are safe for your​ baby." B. ​"Inhalation anesthetics are safe because they remain in your lungs and will not affect the​ baby." C. ​"You will most likely receive local anesthesia so this will not affect your​ baby." D. ​"You will most likely have an epidural so there is no risk to your​ baby."

C. ​"You will most likely receive local anesthesia so this will not affect your​ baby."

A client prescribed valproic acid​ (Depakene) for seizure control is planning a pregnancy. Which statements should the nurse include in a discussion with the​ client? Select all that apply. A. ​"Since your epilepsy may be exacerbated by a​ pregnancy, your healthcare provider will most likely prescribe a second antiepileptic​ prescription." B. ​"Thankfully, the current generation of antiepileptic prescriptions will not interfere with you getting​ pregnant." C. ​"Your current antiepileptic prescription should not be used when you are​ pregnant." D. ​"Folic acid supplementation is important for​ you." E. ​"You should consider adopting a baby instead since there are so many problems associated with epilepsy and​ pregnancy."

C. ​"Your current antiepileptic prescription should not be used when you are​ pregnant." D. ​"Folic acid supplementation is important for​ you."

Which statement is accurate when prescribing acetylsalicylic acid​ (aspirin) for the treatment​ pain? A. Increase consumption of herbs such as garlic and ginger to potentiate the​ anti-inflammatory effects. B. High doses are necessary​ (1 g) to achieve anticoagulant effects. C. ​Enteric-coated capsules are available to reduce GI side effects. D. Low doses​ (325 mg), can significantly reduce inflammation.

C. ​Enteric-coated capsules are available to reduce GI side effects.

A nurse is working with a student nurse. The nurse explains what a client would experience if the adrenergic receptors are activated. Which of the following comments by the student nurse indicates that the education was​ effective? A. Bronchial constriction. B. Increased blood volume. C. ​Fight-or-flight effects. D. ​Rest-and-digest effects.

C. ​Fight-or-flight effects.

A nurse is providing teaching to a client who has a prescription for buspirone. Which of the following instructions should the nurse include? A. Take the drug with grapefruit juice to increase absorption B. Use the drug as needed for anxiety C. Allow 2 to 4 weeks for full therapeutic effects D. Take the drug on an empty stomach

C. Allow 2 to 4 weeks for full therapeutic effects

A nurse is providing teaching to a client who has a prescription for lithium carbonate about reducing the risk for lithium toxicity. Which of the following instructions should the nurse include? A. Consume a low-sodium diet B. Reduce fluid intake C. Avoid taking NSAIDs D. Take the drug with food

C. Avoid taking NSAIDs

A client prescribed onabotulinumtoxinA​ (Botox) for muscle spasms states to the nurse​ "This prescription will also make my face look​ younger." Which information should the nurse provide the​ client Select all that apply... A. ​"Once you start on the​ medication, it may take a week or so before you notice a change in your​ skin." B. ​"Be certain you take the medication with a full glass of water because it can be hard on your​ kidneys." C. ​"There are many different uses for that​ drug, depending on how it is​ administered." D. ​"You may need additional treatments with the medication in a few​ months." E. ​"You should be aware that side effects of the medication can occur hours or weeks after your​ treatment."

C. D. and E.

A nurse is providing teaching to a client about the adverse effects of drug therapy. The nurse should include that orthostatic hypotension is a common adverse reaction of which of the following drugs? A. Venlafaxine B. Bupropion C. Imipramine D. Valproic acid

C. Imipramine

A nurse is caring for a client who is taking venlafaxine to treat major depressive disorder. The nurse should identify that which of the following drugs can cause serotonin syndrome when taken concurrently with venlafaxine? A. Alprazolam B. Phenytoin C. Phenelzine D. Pilocarpine

C. Phenelzine

A nurse is caring for a client who is taking lithium carbonate to treat bipolar disorder. Which of the following diagnostic tests should the nurse recommend that the client undergo periodically? A. Chest x-ray B. Tonometry C. Thyroid function test D. Endoscopic retrograde cholangiopancreatography (ERCP)

C. Thyroid function test

A client prescribed cyclobenzaprine​ (Amrix) one week prior for the treatment of neck spasms reports the medication is not working. Which statements should the nurse provide the​ client? Select all that apply... A. ​"You will need to increase the frequency of the​ dosage." B. ​"You should discontinuation the​ prescription." C. ​"It is important you continue to take the​ prescription." D. ​"You may require a different​ prescription." E. ​"Let's discuss some nonpharmacological techniques that can be used with the​ prescription.

C. and E.

Which client should the nurse be most concerned with taking​ benzodiazepines? A. A​ 9-year-old child with panic attacks. B. A​ 32-year-old mother of two preschool children. C. An​ 87-year-old client who uses a cane for ambulation. D. A​ 42-year-old businessman who travels internationally.

C.An​ 87-year-old client who uses a cane for ambulation.

A client asks the nurse what a​ "physical addiction to​ alcohol" means. Which response should the nurse provide the​ client? A. ​"The brain remembers the euphoria experienced with​ alcohol, and there is a craving for​ it." B. ​"There is an intense craving for​ alcohol, but the craving can best managed through group​ therapy." C. ​"Feelings of depression occur if the use of alcohol is​ stopped." D. ​"The body becomes used to alcohol and if it is abruptly stopped withdrawal symptoms​ occur."

D

A client states to the​ nurse, "I used to drink a pint of whiskey every day. Now I get sick and pass out after just two​ drinks." Which should the nurse be concerned​ with? A. The patient is in denial and is most likely minimizing the amount of alcohol he or she consumes. B. Pancreatic damage C. The patient has brain damage from the alcohol and cannot remember how much he or she consumes. D. Liver damage

D

The nurse is caring for a client suspected of an opioid overdose. Which prescription should the nurse anticipate​ administering? A. Bupropion​ (Zyban) B. Methylphenidate​ (Ritalin) C. Methadone​ (Dolophine) D. Naloxone​ (Narcan)

D

Which drug is least likely to produce physical dependence or​ tolerance? A. Ethyl alcohol B. Morphine C. Nicotine D. Marijuana

D

Which phenomena occurs when a client with a history of alcoholism requires higher than usual amounts of opioids for pain​ relief? A. Resistance to opioids B. Physical dependence C. Paradoxical reaction D. Cross tolerance

D

Which prescription should the nurse anticipate administering to a client experiencing acute alcohol​ withdrawal? A. Disulfiram​ (Antabuse) Your answer is not correct. B. Naltrexone​ (ReVia, Vivitrol) C. Varenicline​ (Chantix) D. Diazepam​ (Valium)

D

Which schedule does methcathinone ​(Cat​) ​belong? A. III B. IV C. II D. I

D

The nurse observes the repeated pattern of muscle contraction of​ client's leg for 5 seconds followed by 2 seconds of relaxation. Which terminology should the nurse use to document the​ finding? A. Dystonia B. Tonic spasm C. Spasticity D. Clonic spasm

D.

Which client experiencing muscle spasms is​ direct-acting antispasmodic medication dantrolene sodium​ (Dantrium) contraindicated? A. A​ 20-year-old with a spinal cord injury. B. A​ 65-year-old with a cerebral vascular accident. C. A​ 40-year-old with multiple sclerosis. D. A​ 57-year-old with congestive heart failure.

D.

Which common adverse effect of cyclobenzaprine​ (Amrix) should the nurse include when educating the​ client? A. Tongue swelling B. Alopecia C. Hypotension D. Drowsiness

D.

The nurse is caring for a client suspected of having bipolar disorder. Which assessment findings that have occurred over the period of a week support the​ nurse's suspicion? A. Difficulty​ sleeping, obsession with​ death, hallucinations. B. ​Delusions, unkempt​ appearance, fatigue. C. Increased​ goal-directed behavior and​ talkativeness, distractibility. D. Abnormal eating​ patterns, feelings of​ despair, flight of ideas.

D. Abnormal eating​ patterns, feelings of​ despair, flight of ideas.

Which statement best explains the action of the prescriptions used to treat​ attention-deficit/hyperactivity disorder​ (ADHD)? A. Certain areas of the brain are​ deactivated, resulting in a calming effect. B. Neurotransmitters are​ blocked, limiting the effects they can produce within the PNS. C. Neurotransmitter levels are​ reduced, which produces a calming effect within the CNS. D. Activation of certain areas of the brain causes increased attention and ability to focus.

D. Activation of certain areas of the brain causes increased attention and ability to focus.

The nurse observes a client with extrapyramidal symptoms having difficulty relaxing. Which should the nurse record in the​ client's record? A. Tardive dyskinesia B. Dystonia C. Secondary parkinsonism D. Akathisia

D. Akathisia

Which statement is accurate regarding​ attention-deficit/hyperactivity disorder​ (ADHD)? A. ADHD is generally diagnosed later in life. B. ADHD is characterized by periods of mania and periods of depression. C. There is a higher incidence of ADHD in girls than in boys. D. Anxiety and social withdrawal are more frequently seen in girls than in boys.

D. Anxiety and social withdrawal are more frequently seen in girls than in boys.

Which should the nurse recognize are the traditionally prescribed drug classifications used to treat​ attention-deficit/hyperactivity disorder​ (ADHD)? A. CNS depressants B. Parasympathomimetics C. Sympathomimetics D. CNS stimulants

D. CNS stimulants

Which statement best describes the contributing factor to the structural change that occurs in the brain of a patient with​ Alzheimer's disease? A. Cerebral bleeding and associated hypoxia B. Increases in blood pressure and cholesterol levels C. Increased acetylcholine levels D. Chronic inflammation and oxidative cellular damage

D. Chronic inflammation and oxidative cellular damage

A parent asks the nurse why his or her​ 8-year-old child cannot have cough syrup with codeine to help the child rest. Prior to responding to the​ parent, which should the nurse​ understand? A. Children are more likely to have an allergic reaction to the codeine. B. Codeine will exacerbate the​ child?s condition. C. The child can become addicted to the cough syrup. D. Codeine is contraindicated in children.

D. Codeine is contraindicated in children.

Which should the nurse recognize is the benefit of a prescription for levodopa for a client with​ Parkinson's disease? A. Can be administered intravenously. B. Has fewer adverse effects. C. Less risk for addiction. D. Crosses the​ blood-brain barrier.

D. Crosses the​ blood-brain barrier.

Which prescription should the nurse anticipate will most likely prescribed for a client with​ Alzheimer's disease? A. Haloperidol​ (Haldol) B. Levodopa​ (Larodopa) C. Benztropine mesylate​ (Cogentin) D. Donepezil hydrochloride​ (Aricept)

D. Donepezil hydrochloride​ (Aricept)

The educator is discussing the symptoms of schizophrenia with a nurse. Which receptor site should the educator associate with the​ symptoms? A. Adrenergic B. Beta 2 C. Cholinergic D. Dopamine type 2

D. Dopamine type 2

A client who has myasthenia gravis​ (MG) presents to the emergency department with abrupt onset of increased muscle weakness and difficulty swallowing. An attempt to distinguish worsening of the MG symptoms from overdose of the​ client's prescribed anticholinergic is planned. What medications should the nurse obtain for use in this​ procedure? Select all that apply. A. Donepezil​ (Aricept) B. Pyridostigmine​ (Mestinon) C. Neostigmine​ (Prostigmin) D. Edrophonium​ (Edrophonium Injectable) E. Atropine​ (Atropine Injectable)

D. Edrophonium​ (Edrophonium Injectable) E. Atropine​ (Atropine Injectable)

The nurse is preparing to review the records for a client with petit mal seizures. Which prescription should the nurse anticipate the client has been​ prescribed? A. Diazepam​ (Valium) B. Carbamazepine​ (Tegretol) C. Phenytoin​ (Dilantin) D. Ethosuximide​ (Zarontin)

D. Ethosuximide​ (Zarontin)

A client presents to the emergency department suspected of overdosing on lorazepam​ (Ativan). Which prescription should the nurse prepare to​ administer? A. Pralidoxime​ (Protopam) B. Nalmefene​ (Revex) C. Naloxone​ (Narcan) D. Flumazenil​ (Romazicon)

D. Flumazenil​ (Romazicon)

A client is prescribed a drug that activates alpha2 receptors. Which physiological response should anticipate when plan of care for the​ client? A. Increased lipolysis. B. Absence of monoamine oxidase. C. Destruction of presynaptic nerve terminals. D. Inhibition of norepinephrine release.

D. Inhibition of norepinephrine release.

The client is quadriplegic and receives the anticholinergic drug oxybutynin​ (Ditropan) to increase his bladder capacity. What is an important assessment of this client by the​ nurse? A. Is he gaining​ weight? B. Is he​ lethargic? C. Is he​ irritable? D. Is he​ constipated?

D. Is he​ constipated?

The​ health-care provider orders dicyclomine​ (Bentyl), an anticholinergic​ drug, for a client. What is the​ nurse's priority assessment prior to administering this​ drug? A. Does the client have light​ sensitivity? B. Is the client dizzy upon​ standing? C. Does the client have a history of​ alcoholism? D. Is the client able to​ urinate?

D. Is the client able to​ urinate?

The nurse is caring for a client in Stage 2 of general anesthesia. Which is a priority nursing​ intervention? A. Assist the anesthesiologist in repositioning the client. B. Insert the indwelling urinary catheter. C. Complete the surgical scrub. D. Keep the environment quiet and calm.

D. Keep the environment quiet and calm.

The nurse is preparing to assist with the repair of a laceration on a client with a history of a cardiovascular disorder. Which anesthesia should the nurse anticipate​ prescribed? A. Lidocaine B. Lidocaine gel C. Benzocaine D. Lidocaine with epinephrine

D. Lidocaine with epinephrine

Which should the nurse monitor the client for that has received a volatile liquid inhalation​ anesthesia? A. Increased intracranial pressure B. Hypertension C. Pneumonia D. Malignant hyperthermia

D. Malignant hyperthermia

Which should the nurse include in the plan of care for the patient with​ Parkinson's disease? A. Limit exercise to decrease the possibility of fractures. B. Monitor the client for psychotic symptoms. C. Check peripheral circulation for thrombophlebitis. D. Monitor the client for the ability to chew and swallow.

D. Monitor the client for the ability to chew and swallow.

The nurse has administered intravenous​ (IV) diazepam​ (Valium) for the client in status epilepticus. Which initial assessment should the nurse​ perform? A. Level of consciousness B. Blood pressure C. Heart rate D. Respirations

D. Respirations

An older adult is prescribed phenobarbital​ (Luminal) for seizure control. Which assessment should the nurse recognize is the most​ important? A. Nutritional status B. Fluid intake C. Electrolyte balance D. Respiratory function

D. Respiratory function

Which is the priority nursing assessment for the client that has received succinylcholine​ (Anectine)? A. Anaphylactic shock B. Spontaneous bleeding C. Delirium D. Respiratory paralysis

D. Respiratory paralysis

The nurse is preparing to assess a client in Stage 3 of general anesthesia. Which assessment findings should the nurse​ anticipate? A. Loss of general sensation during consciousness B. Temporary smooth muscle paralysis C. Irregular heart and respiratory rate D. Respiratory stabilization and slow eye movements

D. Respiratory stabilization and slow eye movements

Which condition is a client most likely experiencing that tells the nurse he or she is having difficulty sleeping because he or she has two final exams​ approaching? A. ​Obsessive-compulsive disorder B. Social anxiety C. Performance anxiety D. Situational anxiety

D. Situational anxiety

The nurse notes that a client prescribed carteolol​ (Cartrol), routinely takes metaproterenol​ (Alupent). Which should the nurse be concerned​ with? A. The client is at risk for a hypertensive crisis. B. The effects of metaproterenol​ (Alupent) will be increased. C. The drugs are​ compatible; there will not be any adverse effects. D. The effects of metaproterenol​ (Alupent) will be decreased.

D. The effects of metaproterenol​ (Alupent) will be decreased.

A client presents to the clinic with symptoms of hyperthermia and diaphoresis. The​ client's prescription history includes escitalopram​ (Lexapro), St.​ John's Wort,​ levothyroxine, and acetaminophen. Which should the nurse suspect has​ occurred? A. The patient may have contracted a virus. B. The patient may be experiencing thyroid toxicosis. C. The patient may be experiencing withdrawal. D. The patient may be experiencing serotonin syndrome.

D. The patient may be experiencing serotonin syndrome.

Which should the nurse recommend to a client as a primary approach to managing​ stress? A. A combined approach using a prescription and nonpharmacological strategies. B. The practice of meditation. C. The use of an anxiolytic. D. To determine the cause of the anxiety

D. To determine the cause of the anxiety

A client is prescribed trihexyphenidyl​ (Artane). Which assessment finding should the nurse report to the healthcare​ provider? A. Dry mouth B. Anorexia C. Hypertension D. Urinary retention

D. Urinary retention

A postoperative client has a prescription for morphine 2 mg IV push every 2 hours and hydrocodone 5 mg every 6 hours. The client received the morphine 2 hours ago and is requesting hydrocodone. Which initial action should the nurse​ take? A. Administer 5 mg of hydrocodone. B. Administer another dose of morphine. C. Encourage the use distraction techniques. D. Using a pain scale assess the​ patient's level of pain.

D. Using a pain scale assess the​ patient's level of pain.

The nurse is preparing to assess a client with multiple sclerosis. Which assessment finding should the nurse​ anticipate? A. Muscle rigidity B. Tremors C. Skeletal muscle atrophy D. Visual disturbances

D. Visual disturbances

The nurse is providing information about seizures to a client. Which statement should the nurse include in the​ discussion? A. ​"Convulsions are a symptom of the underlying seizure​ disorder." B. ​"Epilepsy is an acute disorder characterized by nonconvulsive​ seizures." C. ​"All seizures involve​ convulsions." D. ​"Bacterial infections can cause​ seizures."

D. ​"Bacterial infections can cause​ seizures."

The nurse has provided teaching for a client prescribed viscous lidocaine for oral ulcerations. Which statement made by the client indicates further teaching is​ necessary? A. ​"I will not chew or eat anything until I regain​ feeling." B. ​"I can use a mouthwash after the use of the​ prescription." C. ​"I will swish the prescription around my mouth before spitting it​ out." D. ​"I can drink some water after I swallow the​ prescription."

D. ​"I can drink some water after I swallow the​ prescription."

The nurse has provided a client education on the pharmacologic management of epilepsy. Which statement made by the client indicates an understanding of the​ treatment? A. ​"I will remain on an antiseizure prescription for 1 year after my last​ seizure." B. ​"I will be on a ketogenic diet combined with an antiseizure​ prescription." C. ​"I will avoid foods with tyramine while taking my antiseizure​ prescription." D. ​"I understand that I will take an antiseizure prescription​ indefinitely."

D. ​"I understand that I will take an antiseizure prescription​ indefinitely."

The nurse has provided education for a client prescribed opioid treatment. Which statement made by the client indicates an understanding of the​ education? A. ​"I will make sure I take stool​ softeners." B. ​"I will use a laxative if I need​ to." C. ​"I will increase my physical​ activity." D. ​"I will increase my fluid​ intake."

D. ​"I will increase my fluid​ intake."

The educator is preparing to discuss the actions of succinylcholine​ (Anectine) with a nurse. Which information should the educator​ include? A. ​"It causes the patient to rapidly lose​ consciousness." B. ​"It increases cardiac output by raising the heart​ rate." C. ​"It interferes with impulse​ transmission, resulting in total anesthesia from the pain of​ surgery." D. ​"It reduces the amount of general anesthetic needed for​ procedures."

D. ​"It reduces the amount of general anesthetic needed for​ procedures."

A preoperative client will be receiving glycopyrrolate​ (Robinul), which is an anticholinergic drug. The client asks the​ nurse, "Why do I need to take that​ medication?" What is the best response by the​ nurse? A. ​"It will help maintain your blood pressure during​ surgery." B. ​"It will increase your urinary output during​ surgery." C. ​"It will help you breathe better during​ surgery." D. ​"It will decrease your respiratory secretions during​ surgery."

D. ​"It will decrease your respiratory secretions during​ surgery."

A client that experienced head trauma asks the nurse why they cannot have an opioid analgesic. Which response should the nurse​ provide? A. ​"You may not be able to notice any changes you are​ experiencing." B. ​"Opioids are not effective for the management of pain associated with head​ trauma." C. ​"The use of an opioid analgesics can significantly decrease blood​ pressure." D. ​"Opioids can mask changes in the level of​ consciousness."

D. ​"Opioids can mask changes in the level of​ consciousness."

The nurse is providing education to the family member of a client with​ Alzheimer's disease. Which information should the nurse include in the​ discussion? A. ​"Symptoms should begin improving within a few​ days." B. ​"The prescription have serious side effects if used for a long​ time." C. ​"Symptoms will improve as long as the prescription is taken​ regularly." D. ​"The prescription may help with the symptoms for a little​ while."

D. ​"The prescription may help with the symptoms for a little​ while."

A client asks the nurse to describe NREM stage 3 sleep. Which response should the nurse provide the​ client? A. ​"This is the lightest stage of sleep and is profoundly affected by​ anxiety." B. ​"Dreaming occurs​ here; without dreams you will be irritable and​ paranoid." C. ​"This is the also called paradoxical​ sleep." D. ​"This is the deepest stage of​ sleep; without it you will be tired and​ depressed."

D. ​"This is the deepest stage of​ sleep; without it you will be tired and​ depressed."

A nurse is providing teaching to a client who has a new prescription for diazepam for anxiety disorders. Which of the following client statements indicates an understanding of the teaching? A. "It is not a big deal if I forget my pills for a couple of days." B. "I should limit alcohol intake to one drink daily while taking this medication." C. "I will avoid salt because this medication can increase my blood pressure." D. "If i become pregnant, it is important to let my health care provider know."

D. "If i become pregnant, it is important to let my health care provider know."

A nurse is caring for a client who recently began taking chlorpromazine to treat schizophrenia and who was admitted to the emergency room with spasms of their face and back. Which of the following adverse reactions should the nurse suspect? A. Cholinergic crisis B. Serotonin syndrome C. Stevens-Johnson syndrome D. Acute dystonia

D. Acute dystonia

A nurse is providing teaching for a client who has a new prescription for betaxolol eye drops. Which of the following adverse effects should the nurse include in the teaching? A. Constricted pupils B. Discoloration of the iris C. Hypertension D. Bradycardia

D. Bradycardia

A nurse is caring for a client who was admitted to the emergency department with a head injury. The nurse notes that the client has an existing prescription for bupropion hydrochloride to treat depression. For which of the following adverse effects is the client at increased risk? A. Ischemic stroke B. Drowsiness C. Respiratory depression D. Seizure activity

D. Seizure activity

A nurse is caring for a client who has been taking alprazolam for an extended period of time to treat anxiety. The nurse should identify that abruptly stopping alprazolam therapy can result in which of the following adverse effects? A. Anterograde amnesia B. Respiratory depression C. Paradoxical reaction D. Withdrawal symptoms

D. Withdrawal symptoms

a nurse is teaching a client who is taking allopurinol about minimizing adverse effects. Which of the following instructions should the nurse include? eat a small meal before taking the drug suck on hard candy or chew gum take a stool softener daily avoid the use of NSAIDs

eat a small meal before taking the drug

A nurse is caring for a client who has a prescription for alprazolam. For which of the following adverse effects should the nurse monitor? (Select all that apply.) Tolerance Anxiety Sedation Respiratory depression Constipation

Tolerance Anxiety Sedation Respiratory depression

A nurse is caring for a client who is about to begin taking lithium carbonate to treat bipolar disorder. The nurse should instruct the client to monitor for which of the following findings as indications of lithium toxicity? (Select all that apply.) Tremors Confusion Bronchospasm Nausea Muscle weakness

Tremors Confusion Nausea Muscle weakness

a nurse is reviewing the drug list for a client who has a new prescription for allopurinol. The nurse should identify that which of the following drug reacts with allopurinol? Warfarin Ibuprofen Insulin Furosemide

Warfarin

a nurse is reviewing the medical record of a client who reports taking acetaminophen at home. The nurse should identify that which of the following client conditions is contraindicated for acetaminophen? asthma diabetes mellitus heart failure alcohol use disorder

alcohol use disorder

a nurse is caring for a child who has a viral infection. The nurse should identify that which of the following drugs can increase the risk of Reye syndrome in children who have viral infections? butorphanol acetaminophen tramadol aspirin

aspirin

a nurse is caring for a client who has a new prescription for prednisone for long-term treatment of rheumatoid arthritis. The nurse should monitor the client for which of the following adverse drug reactions? pulmonary embolism hepatitis bone loss breast cancer

bone loss

a nurse is reviewing the medical record of a client who has a new prescription for tramadol. The nurse should identify that which of the following conditions is a contraindication for tramadol? hyperthyroidism seizure disorder rheumatoid arthritis urinary incontinence

seizure disorder

a nurse is caring for a client who has a new prescription for celecoxib. The nurse should tell the client to report which of the following adverse drug reactions? tinnitus chest pain constipation diaphoresis

chest pain

a nurse is caring for a client who is taking acetaminophen at regular intervals for mild discomfort. The nurse should tell the client to report which of the following early indications of acetaminophen toxicity? (Select all that apply.) diaphoresis palpitations shortness of breath nausea diarrhea

diaphoresis nausea diarrhea

a nurse is caring for a client who is taking allopurinol to treat gout. The nurse should monitor the client for which of the following manifestations of hypersensitivity syndrome? muscle pain fever anxiety tremors

fever

a nurse is caring for a client who is about to begin taking aspirin. The nurse should instruct the client to report which of the following manifestations of salicylism? (select all that apply) fever tinnitus diaphoresis thrombophlebitis dizziness

fever tinnitus diaphoresis dizziness

a nurse is caring for a client who currently takes furosemide and has a new prescription for prednisone. The nurse should monitor the client for which of the following manifestations during concurrent use of the drugs? hypercalcemia hypoglycemia hypothermia hypokalemia

hypokalemia

a nurse is teaching a client who has a new prescription for tramadol. Which of the following instructions should the nurse include? (select all that apply) increase fiber and fluid intake take the drug with food avoid driving after taking the drug change positions gradually reduce exercise level temporarily

increase fiber and fluid intake take the drug with food avoid driving after taking the drug change positions gradually

a nurse is caring for a client who is taking naloxone to treat acute morphine toxicity. The nurse should monitor the client for which of the following drug reactions? (select all the apply) increased respiratory rate increased pain thromboplebitis ventricular arrhythmias hypertension

increased respiratory rate increased pain ventricular arrhythmias hypertension

a nurse is planing care for a client who has started taking prednisone. Which of the following interventions should the nurse include? monitor the clients blood glucose administer an antacid 30 min prior to prednisone administer aspirin rather than NSAIDs if the client has pain monitor the client for hyperkalemia

monitor the clients blood glucose

a nurse is caring for a client who has a new prescription for butorphanol. The nurse should monitor the client for which of the following adverse drug reactions? (select all that apply) infection nausea tachycardia dizziness headache

nausea dizziness headache

a nurse is teaching s client who has a new prescription for allopurinol. The nurse should instruct the client to report which of the following adverse drug reactions? (select all that apply) palpitations sore throat vertigo bruising vision changes

sore throat vertigo bruising vision changes

a nurse is reviewing the medical record of a client who has a new prescription for celecoxib. The nurse should identify that which of the following conditions is a contraindication to celecoxib? rheumatoid arthritis ankylosing spondylitis sulfonamide allergy adrenocortical insufficiency

sulfonamide allergy

a nurse is caring for a client who asks about taking acetaminophen. The nurse should identify that acetaminophen is indicated for which of the following conditions? (select all that apply) to reduce fever to decrease inflammation to relieve pain to promote sedation to alleviate anxiety

to reduce fever to relieve pain

a nurse is caring for a client who is reeivng morphine to relieve severe pain. The nurse should monitor the client for which of the following adverse drug reactions? (select all the apply) diarrhea urinary retention respiratory depression sedation orthostatic hypotension

urinary retention respiratory depression sedation orthostatic hypotension

a nurse is caring for a client who is opioid dependent and has a new prescription for butorphanol. The nurse should monitor the client for which of the following manifestations of abstinence syndrome? (select all that apply) bronchospasm vomiting peripheral edema abdominal cramps hypertension

vomiting abdominal cramps hypertension

a nurse is caring for a client who has a prescription for aspirin to treat an ankle pain. The nurse should instruct the client to report which of the following adverse drug reactions? polyuriabone pain weight gain infection

weight gain


संबंधित स्टडी सेट्स

Ch 14 - Head, Face, Neck, and Regional Lymphatics

View Set

Statistical Process Control (SPC) CONTROL

View Set

American Revolution after Declaration of Independence

View Set

Chapter 3, 5, & 6 test study guide

View Set

Traumapedia and orthobullets Hand fractures and dislocations

View Set

Com 317 - Chapter 12 (Relevant costs for Decision Making)

View Set